Google Groups no longer supports new Usenet posts or subscriptions. Historical content remains viewable.
Dismiss

So-called "centrifugal force"

6 views
Skip to first unread message

ce...@achilles.net

unread,
Apr 30, 1997, 3:00:00 AM4/30/97
to

Webster's 9th New Collegiate Dictionary defines "centrifugal force" on
page 221 as the force that an object moving along a circular path
exerts on the body constraining the object and that acts outwardly
away form the centre of rotation. eg: a stone whirled on a string
exerts centrifugal force on the string

Since when?!?! I was under the impression that the only acceleration
on that stone/string problem was centripetal acceleration tangent to
the circle in which the stone travels!

(I was also under the impression that "centrifugal force" was made up
to describe phenomena, and never existed)

So, (and this sounds like a silly question) who's right: me, or
Webster's Dictionary <grin>?


Peter Diehr

unread,
Apr 30, 1997, 3:00:00 AM4/30/97
to

I...@Ebon.Psi <ce...@achilles.net> wrote in article
<5k7cn6$j...@hermes.achilles.net>...

"Centrifugal force" is a so-called "inertial force" or
"pseudo-force" which is introduced into the accelerating frame in
order to make Newton's Laws of Motion work.

The effects are real (you feel them), but to the observer in an
inertial frame of reference, it is clear that they are not
fundamental forces.

Best Regards, Peter


Lawrence R. Mead

unread,
Apr 30, 1997, 3:00:00 AM4/30/97
to

I...@Ebon.Psi (ce...@achilles.net) wrote:
: Webster's 9th New Collegiate Dictionary defines "centrifugal force" on

: page 221 as the force that an object moving along a circular path
: exerts on the body constraining the object and that acts outwardly
: away form the centre of rotation. eg: a stone whirled on a string
: exerts centrifugal force on the string
:
: Since when?!?! I was under the impression that the only acceleration
: on that stone/string problem was centripetal acceleration tangent to
: the circle in which the stone travels!

Almost. Centripetal means center seeking; the centripetal force - the
tension in the string - is radially *inward*. The dictionary, of course,
is flat wrong.

--

Lawrence R. Mead (lrm...@whale.st.usm.edu)
ESCHEW OBFUSCATION ! ESPOUSE ELUCIDATION !
http://www-dept.usm.edu/~scitech/phy/mead.html

Wayne Throop

unread,
Apr 30, 1997, 3:00:00 AM4/30/97
to

: ce...@achilles.net (I...@Ebon.Psi)

: Webster's 9th New Collegiate Dictionary defines "centrifugal force" on
: page 221 as the force that an object moving along a circular path
: exerts on the body constraining the object and that acts outwardly
: away form the centre of rotation. eg: a stone whirled on a string
: exerts centrifugal force on the string
:
: Since when?!?! I was under the impression that the only acceleration
: on that stone/string problem was centripetal acceleration tangent to
: the circle in which the stone travels!

You've changed the subject from "force" to "acceleration".
True, the only *acceleration* (if considered in an inertial frame)
is a centripetal acceleration. But they were talking about forces.

Consider where the string meets the stone.
The string exerts a force on the stone (to make it accelerate). That's
the centripetal force. By newtonian "every action has an equal and
opposite reaction" (in this case, every force), the stone is also
exerting a force on the string. This "reaction force" is called "the
centrifugal force". It's a really-for-true force, from the viewpoint of
the bit of glue that holds the string to the stone.

: (I was also under the impression that "centrifugal force" was made up


: to describe phenomena, and never existed)

That's a more formal use of "centrifugal force". It's a pseudoforce (or
less prejudicially, "inertial force") that's introduced if you are using
a non-inertial frame. A frame in which the string and rock are not
moving at all, so that you can analyze the situation statically.

: So, (and this sounds like a silly question) who's right: me, or


: Webster's Dictionary <grin>?

Both. The term "centrifugal force" is used to describe both the
reaction force to the centripetal force (in inertial coordinates),
and for the inertial force (introduced in rotating coordinates).

Language is funny that way. Same words get used for multiple referents.

--
Wayne Throop thr...@sheol.org http://sheol.org/throopw
thr...@cisco.com

Dan Evens

unread,
Apr 30, 1997, 3:00:00 AM4/30/97
to

I...@Ebon.Psi wrote:
>
> Webster's 9th New Collegiate Dictionary defines "centrifugal force" on
> page 221 as the force that an object moving along a circular path
> exerts on the body constraining the object and that acts outwardly
> away form the centre of rotation. eg: a stone whirled on a string
> exerts centrifugal force on the string
>
> Since when?!?! I was under the impression that the only acceleration
> on that stone/string problem was centripetal acceleration tangent to
> the circle in which the stone travels!

Centripetal force is the force the "string" applies to the mass.
Obviously it is straight along the "string" (neglecting any tiny
wind resistance on the string). (The term "string" here is to mean
whatever is pulling the mass into a circle.)

Centrifugal force is a ficitious force which is observed because
the observer riding on the mass assumes s/he is at rest. So, if
you are sitting there, you feel the mass pushing into your butt.
Just as you assume when you are sitting at your desk where you
feel the Earth pulling you down, so you feel the cicular motion
pulling you down. In actual fact, in both cases, the force you
feel is the thing you are sitting on pushing you off the path
you would follow if nothing was pusing you around.

If the circular motion is continuing at the same angular velocity
then there will not be any tangential acceleration.

> (I was also under the impression that "centrifugal force" was made up
> to describe phenomena, and never existed)
>

> So, (and this sounds like a silly question) who's right: me, or
> Webster's Dictionary <grin>?

Neither of you did a spectacularly good job of describing things.
Dan Evens

me...@cars3.uchicago.edu

unread,
Apr 30, 1997, 3:00:00 AM4/30/97
to

In article <5k7cn6$j...@hermes.achilles.net>, ce...@achilles.net (I...@Ebon.Psi) writes:
>Webster's 9th New Collegiate Dictionary defines "centrifugal force" on
>page 221 as the force that an object moving along a circular path
>exerts on the body constraining the object and that acts outwardly
>away form the centre of rotation. eg: a stone whirled on a string
>exerts centrifugal force on the string
>
>Since when?!?! I was under the impression that the only acceleration
>on that stone/string problem was centripetal acceleration tangent to
>the circle in which the stone travels!
>

Depends on your reference frame.

>(I was also under the impression that "centrifugal force" was made up
>to describe phenomena, and never existed)
>

Yeah. Generations upon generations of teachers perpetuate this
nonsense. Centrifugal force is perfectly real in the sense that it
does provide adequate description of motion reltive to accelerated
frames of reference. It is referred to as "fictitious" since it can be
made to "disappear" by an appropriate geometrical transformation, i.e.
there is no actual physical mechanism (describable in terms of forces
between particles) that gives rise to it. Nevertheless, its effects,
when describing motion relative to a rotating frame, are very much
real.

Mati Meron | "When you argue with a fool,
me...@cars.uchicago.edu | chances are he is doing just the same"

Ken Fischer

unread,
May 1, 1997, 3:00:00 AM5/1/97
to

I...@Ebon.Psi (ce...@achilles.net) wrote:
: Webster's 9th New Collegiate Dictionary defines "centrifugal force" on

: page 221 as the force that an object moving along a circular path
: exerts on the body constraining the object and that acts outwardly
: away form the centre of rotation. eg: a stone whirled on a string
: exerts centrifugal force on the string

: Since when?!?! I was under the impression that the only acceleration
: on that stone/string problem was centripetal acceleration tangent to
: the circle in which the stone travels!

The only "acceleration" is centripetal, but there
is such a thing as "centrifugal force" even if it is only
inertial.

: (I was also under the impression that "centrifugal force" was made up


: to describe phenomena, and never existed)

Within the last 50 years (since I was in school),
there has been a move by some teachers to eliminate the
term "centrifugal force", which seems a bit restrictive.
What will they call a "centrifuge" if they are
successful? :-)

: So, (and this sounds like a silly question) who's right: me, or


: Webster's Dictionary <grin>?

Maybe both are right if the terminology is
straightened out. In the case of the string and the
stone, the acceleration is inward (centripetal), and
a force (inertial) holding the string taunt (centrifugal),
it really isn't a crime to say centrifugal, but it seems
to be out of vogue.


ce...@achilles.net

unread,
May 1, 1997, 3:00:00 AM5/1/97
to

> The only "acceleration" is centripetal, but there
>is such a thing as "centrifugal force" even if it is only
>inertial.

So, what I've been able to glean from your replies is that the
orbiting object (which is accelerating, isn't in an inertial reference
frame, etc, etc) is the only one who thinks centrifugal force is real.

>: So, (and this sounds like a silly question) who's right: me, or
>: Webster's Dictionary <grin>?

> Maybe both are right if the terminology is
>straightened out. In the case of the string and the
>stone, the acceleration is inward (centripetal), and
>a force (inertial) holding the string taunt (centrifugal),
>it really isn't a crime to say centrifugal, but it seems
>to be out of vogue.

Hm. Strange how ridicule for a term that tends to confuse ppl unless
it's explained fully takes the place of a (relatively) simple
explanation. Thanks yall.

Ken Fischer

unread,
May 1, 1997, 3:00:00 AM5/1/97
to

I...@Ebon.Psi (ce...@achilles.net) wrote:
: > The only "acceleration" is centripetal, but there
: >is such a thing as "centrifugal force" even if it is only
: >inertial.

: So, what I've been able to glean from your replies is that the
: orbiting object (which is accelerating, isn't in an inertial reference
: frame, etc, etc) is the only one who thinks centrifugal force is real.

Please don't use my example for gravitation, that
is very different from the string and stone, unless you
want simple Newtonian concepts.

: >: So, (and this sounds like a silly question) who's right: me, or


: >: Webster's Dictionary <grin>?

: > Maybe both are right if the terminology is
: >straightened out. In the case of the string and the
: >stone, the acceleration is inward (centripetal), and
: >a force (inertial) holding the string taunt (centrifugal),
: >it really isn't a crime to say centrifugal, but it seems
: >to be out of vogue.

: Hm. Strange how ridicule for a term that tends to confuse ppl unless
: it's explained fully takes the place of a (relatively) simple
: explanation. Thanks yall.

It never was a problem until "centrifugal" force
became an outcast, and that probably was caused by some
teachers who _didn't_ understand the problem very well.
Many "forces" are inertial forces, and that
doesn't make them any less real, but may change or null
a value in certain reference frames in certain systems
of bookkeeping.
But it is true for the string and stone, the
only "acceleration" is centripetal.

Ken Fischer

John E. Prussing

unread,
May 2, 1997, 3:00:00 AM5/2/97
to

me...@cars3.uchicago.edu writes:

>In article <5k7cn6$j...@hermes.achilles.net>, ce...@achilles.net (I...@Ebon.Psi) writes:

>>Webster's 9th New Collegiate Dictionary defines "centrifugal force" on
>>page 221 as the force that an object moving along a circular path
>>exerts on the body constraining the object and that acts outwardly
>>away form the centre of rotation. eg: a stone whirled on a string
>>exerts centrifugal force on the string
>>
>>Since when?!?! I was under the impression that the only acceleration
>>on that stone/string problem was centripetal acceleration tangent to
>>the circle in which the stone travels!
>>

>Depends on your reference frame.

>>(I was also under the impression that "centrifugal force" was made up


>>to describe phenomena, and never existed)
>>

>Yeah. Generations upon generations of teachers perpetuate this
>nonsense. Centrifugal force is perfectly real in the sense that it
>does provide adequate description of motion reltive to accelerated
>frames of reference. It is referred to as "fictitious" since it can be
>made to "disappear" by an appropriate geometrical transformation, i.e.
>there is no actual physical mechanism (describable in terms of forces
>between particles) that gives rise to it. Nevertheless, its effects,
>when describing motion relative to a rotating frame, are very much
>real.

Exactly. I much prefer the term "apparent" to "fictitious". I think the
terminology causes the problem and the distrust of the term centrifugal
force.

An observer in the rotating frame senses the centrifugal force in
the same way an observer in an upwardly-accelerating elevator senses a
downward force. Centrifugal force is a perfectly valid concept if used
properly. These apparent forces explain motion in non-inertial frames
of reference.

What's a good term for the apparent downward force in an upwardly-
accelerated elevator, opp-acceleration? Translational and rotational
motion deserve equal treatment. This is America, after all.

>Mati Meron | "When you argue with a fool,
>me...@cars.uchicago.edu | chances are he is doing just the same"

--
=-=-=-=-=-=-=-=-=
John E. Prussing
=-=-=-=-=-=-=-=-=

john baez

unread,
May 2, 1997, 3:00:00 AM5/2/97
to

In article <5k7cn6$j...@hermes.achilles.net>,

I...@Ebon.Psi <ce...@achilles.net> wrote:
>Webster's 9th New Collegiate Dictionary defines "centrifugal force" on
>page 221 as the force that an object moving along a circular path
>exerts on the body constraining the object and that acts outwardly
>away form the centre of rotation. eg: a stone whirled on a string
>exerts centrifugal force on the string
>
>Since when?!?! I was under the impression that the only acceleration
>on that stone/string problem was centripetal acceleration tangent to
>the circle in which the stone travels!

You're right (or at least not wrong).

>(I was also under the impression that "centrifugal force" was made up
>to describe phenomena, and never existed)

All concepts of "force" are made up to describe phenomena. If
we work in a nonrotating coordinate system there is no real
need for the concept of centrifugal force. If we work in a
rotating coordinate system and still want F = ma to apply in
a simple-minded manner, we need the concept of centrifugal
force. Now, you could justly say that expecting F = ma to hold
in a rotating coordinate system is a dumb idea. However, we can
get F = ma to hold in a rotating coordinate system if we make
up two forces: the "centrifugal force" and the "Coriolis force".

One sometimes calls a force that's an artifact of a funny choice of
coordinate system a "ficitious force". The "centrifugal force" and
the "Coriolis force" are two famous examples. Another is gravity, but
that's a more complicated story, which one finds under General
Relativity rather than Classical Mechanics.

>So, (and this sounds like a silly question) who's right: me, or
>Webster's Dictionary <grin>?

Both. Webster's dictionary gives definitions but detailed
explanations. For example, I bet it fails to note that gravity
is not really a force, but merely the curvature of spacetime.
You might check to see what it says under "aether" or "phlogiston".

But wait! The 1913 edition of Webster's Dictionary *does* point out
that "phlogiston" is discredited:

Phlogiston, n. [NL., fr. Gr. burnt, set on fire, fr. to set on fire,
to burn, fr. , , a flame, blaze. See Phlox.] (Old Chem.) The hypothetical
principle of fire, or inflammability, regarded by Stahl as a chemical element.
This was supposed to be united with combustible (phlogisticated) bodies and
to be separated from incombustible (dephlogisticated) bodies, the phenomena
of flame and burning being the escape of phlogiston. Soot and sulphur were
regarded as nearly pure phlogiston. The essential principle of this theory
was, that combustion was a decomposition rather than the union and
combination which it has since been shown to be. <-- this theory is now
discredited and superseded by the theory of chemical reaction between
oxidizable substances and oxidants as an explanation of combustion -->

Wow, that *is* a pretty detailed explanation! (I hadn't known that
soot and sulphur were regarded as nearly pure phlogiston!) Somehow
I think more recent editions would give fewer details, as part of
the general dumbing-down (or more precisely postliterate) phase we
find ourselves in, but I don't have any newer editions available.

So, let's look up "centrifugal" in the 1913 edition...

Centrifugal force (Mech.), a force whose direction is from a center.
When a body moves in a circle with uniform velocity, a force must
act on the body to keep it in the circle without change of velocity. The
direction of this force is towards the center of the circle. If this
force is applied by means of a string to the body, the string will be
in a state of tension. To a person holding the other end of the string,
this tension will appear to be directed toward the body as if the body
had a tendency to move away from the center of the circle which it is
describing. Hence this latter force is often called centrifugal force.
The force which really acts on the body being directed towards the center
of the circle is called centripetal force, and in some popular treatises
the centripetal and centrifugal forces are described as opposing and
balancing each other. But they are merely the different aspects of the
same stress.

Whoa! So back then, they *did* explain centrifugal force more
carefully! Okay, so clearly your problem is that you're using a
crappy new edition instead of one of the good old ones. In the
future, unless you are looking for new words, you should probably use
the 1913 edition, available at:

http://humanities.uchicago.edu/forms_unrest/webster.form.html


Mike Norris

unread,
May 3, 1997, 3:00:00 AM5/3/97
to

> In article <E9HJy...@iglou.com>, Ken Fischer
> <URL:mailto:kefi...@iglou.com> wrote:

> >
> >
> > I...@Ebon.Psi (ce...@achilles.net) wrote:
> > : Webster's 9th New Collegiate Dictionary defines "centrifugal force" on
> > : page 221 as the force that an object moving along a circular path
> > : exerts on the body constraining the object and that acts outwardly
> > : away form the centre of rotation. eg: a stone whirled on a string
> > : exerts centrifugal force on the string
> >
> > : Since when?!?! I was under the impression that the only acceleration
> > : on that stone/string problem was centripetal acceleration tangent to
> > : the circle in which the stone travels!
> >
> > The only "acceleration" is centripetal, but there
> > is such a thing as "centrifugal force" even if it is only
> > inertial.
> >
> > : (I was also under the impression that "centrifugal force" was made up

> > : to describe phenomena, and never existed)
> >
> > Within the last 50 years (since I was in school),
> > there has been a move by some teachers to eliminate the
> > term "centrifugal force", which seems a bit restrictive.
> > What will they call a "centrifuge" if they are
> > successful? :-)
> >
> > : So, (and this sounds like a silly question) who's right: me, or

> > : Webster's Dictionary <grin>?
> >
> > Maybe both are right if the terminology is
> > straightened out. In the case of the string and the
> > stone, the acceleration is inward (centripetal), and
> > a force (inertial) holding the string taunt (centrifugal),
> > it really isn't a crime to say centrifugal, but it seems
> > to be out of vogue.
> >
WRONG WRONG WRONG ... thrice WRONG

Us 'modern' teachers have it right!

There are NO outward forces holding the string taut, if there were
then the forces would balance ... there would be NO ACCELERATION.

Dan Evens has it right ... put another way ...

There is a SINGLE centripetally acting force (caused by pulling the string)
not only on the mass but on every part/element of the string ...
a single UNBALANCED force causes an acceleration, which is what is
happening to the string/mass system.

Remember the whole system is
ACCELERATING TOWARDS THE CENTRE POINT!

X centrifuge X ...
... device that contributes lack of centripete to suspended particles!!!!!

There is no case involving circular motion that requires a
fictitious centrigugal explanation!!!!

Or do you know better?

mi...@mikenorr.demon.co.uk
The English Lake District.


me...@cars3.uchicago.edu

unread,
May 3, 1997, 3:00:00 AM5/3/97
to

In article <ant03170...@mikenorr.demon.co.uk>, Mike Norris <mi...@mikenorr.demon.co.uk> writes:
>> In article <E9HJy...@iglou.com>, Ken Fischer

>> > Maybe both are right if the terminology is


>> > straightened out. In the case of the string and the
>> > stone, the acceleration is inward (centripetal), and
>> > a force (inertial) holding the string taunt (centrifugal),
>> > it really isn't a crime to say centrifugal, but it seems
>> > to be out of vogue.
>> >
>WRONG WRONG WRONG ... thrice WRONG
>
>Us 'modern' teachers have it right!
>

Yeah, sure :-)

... snip ...


>
>There is no case involving circular motion that requires a
>fictitious centrigugal explanation!!!!

Depends what you call "require". Any case can, in principle, be
analyzed relative to an inertial frame, in which case you indeed don't
need to consider anything "centrifugal". Often, though, it is more
convenient to analyze a specific case relative to a non inertial frame
and then you do use both centrifugal force and Coriolis force. Any
working phycisist knows that it would be pure stupidity not to used
the most convenient frame for the specific problem at hand. If
teachers don't realize it, too bad.

>Or do you know better?
>

Yes, I do know better.

Mike Norris

unread,
May 3, 1997, 3:00:00 AM5/3/97
to

mi...@mikenorr.demon.co.uk
The English Lake District.
On Fri 02 May, Mike Norris wrote:
> In article <E9HJy...@iglou.com>, Ken Fischer
> <URL:mailto:kefi...@iglou.com> wrote:

> >
> >
> > I...@Ebon.Psi (ce...@achilles.net) wrote:
> > : Webster's 9th New Collegiate Dictionary defines "centrifugal force" on
> > : page 221 as the force that an object moving along a circular path
> > : exerts on the body constraining the object and that acts outwardly
> > : away form the centre of rotation. eg: a stone whirled on a string
> > : exerts centrifugal force on the string
> >
> > : Since when?!?! I was under the impression that the only acceleration
> > : on that stone/string problem was centripetal acceleration tangent to
> > : the circle in which the stone travels!
> >
> > The only "acceleration" is centripetal, but there
> > is such a thing as "centrifugal force" even if it is only
> > inertial.
> >
> > : (I was also under the impression that "centrifugal force" was made up

> > : to describe phenomena, and never existed)
> >
> > Within the last 50 years (since I was in school),
> > there has been a move by some teachers to eliminate the
> > term "centrifugal force", which seems a bit restrictive.
> > What will they call a "centrifuge" if they are
> > successful? :-)
> >
> > : So, (and this sounds like a silly question) who's right: me, or

> > : Webster's Dictionary <grin>?
> >
> > Maybe both are right if the terminology is
> > straightened out. In the case of the string and the
> > stone, the acceleration is inward (centripetal), and
> > a force (inertial) holding the string taunt (centrifugal),
> > it really isn't a crime to say centrifugal, but it seems
> > to be out of vogue.
> >
WRONG WRONG WRONG ... thrice WRONG

Us 'modern' teachers have it right!

There are NO outward forces holding the string taut, if there were


then the forces would balance ... there would be NO ACCELERATION.

Dan Evens has it right ... put another way ...

There is a SINGLE centripetally acting force (caused by pulling the string)
not only on the mass but on every part/element of the string ...
a single UNBALANCED force causes an acceleration, which is what is
happening to the string/mass system.

The string is NOT in a state of tension in the statics use of the term!

Remember the whole system is
ACCELERATING TOWARDS THE CENTRE POINT!

X centrifuge X ...
... device that contributes lack of centripete to suspended particles!!!!!

There is no case involving circular motion that requires a
fictitious centrigugal explanation!!!!

Or do you know better?

mi...@mikenorr.demon.co.uk
The English Lake District.

John E. Prussing

unread,
May 4, 1997, 3:00:00 AM5/4/97
to

me...@cars3.uchicago.edu writes:

>>> In article <E9HJy...@iglou.com>, Ken Fischer

>>> > Maybe both are right if the terminology is


>>> > straightened out. In the case of the string and the
>>> > stone, the acceleration is inward (centripetal), and
>>> > a force (inertial) holding the string taunt (centrifugal),
>>> > it really isn't a crime to say centrifugal, but it seems
>>> > to be out of vogue.
>>> >
>>WRONG WRONG WRONG ... thrice WRONG
>>
>>Us 'modern' teachers have it right!
>>

>Yeah, sure :-)

> ... snip ...
>>


>>There is no case involving circular motion that requires a
>>fictitious centrigugal explanation!!!!

>Depends what you call "require". Any case can, in principle, be

>analyzed relative to an inertial frame, in which case you indeed don't
>need to consider anything "centrifugal". Often, though, it is more
>convenient to analyze a specific case relative to a non inertial frame
>and then you do use both centrifugal force and Coriolis force. Any
>working phycisist knows that it would be pure stupidity not to used
>the most convenient frame for the specific problem at hand. If
>teachers don't realize it, too bad.

>>Or do you know better?
>>


>Yes, I do know better.

Once again, I submit that a translational, rather than rotational, example
of an apparent force in a non-inertial frame is easier to understand,
namely an upwardly accelerating elevator. It illustrates the same concept
as centrifugal force. An observer in the elevator experiences a downward
force, let's call it oppceleration, that is opposite in direction to
the inertial acceleration, just like centrifugal force is opposite in
direction to centripetal force. The oppceleration is an apparent force
that is necessary to describe the motion relative to the (non-inertial)
observer in the frame fixed to the elevator.

Back to the rotating frame. I work in orbital mechanics, and the reason
that an astronaut on the flight deck of the shuttle experiences a "zero-g"
(a misnomer) or "weightless" (another misnomer) environment is that he
or she is in circular orbit (this is "circular motion") in a rotating,
non-inertial frame and the apparent centrifugal force "cancels" the
gravitational force, resulting in an apparent force-free environment,
with stuff floating around the flight deck. The orbiting shuttle with
its on-board tv pictures should be a convincing experiment to prove
the concept. So what's the problem?

=-=-=-=
John E. Prussing
http://www.uiuc.edu/ph/www/prussing/
=-=-=-=

me...@cars3.uchicago.edu

unread,
May 4, 1997, 3:00:00 AM5/4/97
to

In article <5kgobg$a...@vixen.cso.uiuc.edu>, prus...@staff.uiuc.edu (John E. Prussing) writes:
>me...@cars3.uchicago.edu writes:
>
>>In article <ant03170...@mikenorr.demon.co.uk>, Mike Norris <mi...@mikenorr.demon.co.uk> writes:
>>>
>>>There is no case involving circular motion that requires a
>>>fictitious centrigugal explanation!!!!
>
>>Depends what you call "require". Any case can, in principle, be
>>analyzed relative to an inertial frame, in which case you indeed don't
>>need to consider anything "centrifugal". Often, though, it is more
>>convenient to analyze a specific case relative to a non inertial frame
>>and then you do use both centrifugal force and Coriolis force. Any
>>working phycisist knows that it would be pure stupidity not to used
>>the most convenient frame for the specific problem at hand. If
>>teachers don't realize it, too bad.
>
>>>Or do you know better?
>>>
>>Yes, I do know better.
>
>Once again, I submit that a translational, rather than rotational, example
>of an apparent force in a non-inertial frame is easier to understand,
>namely an upwardly accelerating elevator. It illustrates the same concept
>as centrifugal force. An observer in the elevator experiences a downward
>force, let's call it oppceleration, that is opposite in direction to
>the inertial acceleration, just like centrifugal force is opposite in
>direction to centripetal force. The oppceleration is an apparent force
>that is necessary to describe the motion relative to the (non-inertial)
>observer in the frame fixed to the elevator.
>

Right on.

>Back to the rotating frame. I work in orbital mechanics, and the reason
>that an astronaut on the flight deck of the shuttle experiences a "zero-g"
>(a misnomer) or "weightless" (another misnomer) environment is that he
>or she is in circular orbit (this is "circular motion") in a rotating,
>non-inertial frame and the apparent centrifugal force "cancels" the
>gravitational force, resulting in an apparent force-free environment,
>with stuff floating around the flight deck. The orbiting shuttle with
>its on-board tv pictures should be a convincing experiment to prove
>the concept. So what's the problem?

There's no problem. And the astronaut will find it way simpler to
express the motion of objects on the flight deck using inertial
forces, than transforming everything to an inertial frame, performing
the calculations there, then transforming back (in case somebody
doubts that it is simpler, he's welcome to try).

Ron Shepard

unread,
May 4, 1997, 3:00:00 AM5/4/97
to
E. Prussing) wrote:

[...]


> Back to the rotating frame. I work in orbital mechanics, and the reason
> that an astronaut on the flight deck of the shuttle experiences a "zero-g"
> (a misnomer) or "weightless" (another misnomer) environment is that he
> or she is in circular orbit (this is "circular motion") in a rotating,
> non-inertial frame and the apparent centrifugal force "cancels" the
> gravitational force, resulting in an apparent force-free environment,
> with stuff floating around the flight deck. The orbiting shuttle with
> its on-board tv pictures should be a convincing experiment to prove
> the concept. So what's the problem?

If the shuttle were in free fall going straight towards the earth surface,
then things inside the shuttle would float around too. In fact, the
shuttle in a stationary orbit (which is an elipse, not just a circle) is a
special case of free fall. I'm not exactly sure what is the "problem",
but it is inconsistent to describe one free-fall situation in a cartesian
frame with no "extra" forces, but the other one in a polar frame with this
extra "centrifugal" force. Yes, this is perhaps the best way to describe
the two cases, but the two cases are really two instances of the same
free-fall situation. BTW, it seems to me that the centrifugal force
arises from the use of a polar coordinate system, not from a rotating
reference frame. Lots of SR and GR stuff seems to depend on moving and
rotating reference frames, but centrifugal force is not one of them.

$.02 -Ron Shepard

Ken Fischer

unread,
May 4, 1997, 3:00:00 AM5/4/97
to

Mike Norris (mi...@mikenorr.demon.co.uk) wrote objections to my
statements which attempted to explain why Webster's is correct,
but Mike makes it clear that Webster used a poor example, the
stone on a string, because for a person to twirl a stone on a
string, most of the forces are neither centripetal, nor centri-
fugal, as the hand pulls the string in a circle.

My previous remarks were directed toward a single problem,
that of a string attached to a rotating pivot, and only considered
those forces that exist in that case.

I urge all students to follow the teaching of the textbook,
regardless of what it says. I will reply to Mike at the end
of this long message, I would like to reduce the size, but it
isn't possible.

: mi...@mikenorr.demon.co.uk


: The English Lake District.
: On Fri 02 May, Mike Norris wrote:

: > In article <E9HJy...@iglou.com>, Ken Fischer wrote:
: > >

: > > I...@Ebon.Psi (ce...@achilles.net) wrote:
: > > : Webster's 9th New Collegiate Dictionary defines "centrifugal force" on
: > > : page 221 as the force that an object moving along a circular path
: > > : exerts on the body constraining the object and that acts outwardly
: > > : away form the centre of rotation. eg: a stone whirled on a string
: > > : exerts centrifugal force on the string
: > >
: > > : Since when?!?! I was under the impression that the only acceleration
: > > : on that stone/string problem was centripetal acceleration tangent to
: > > : the circle in which the stone travels!

: > >
: > > The only "acceleration" is centripetal, but there

: > > is such a thing as "centrifugal force" even if it is only
: > > inertial.
: > >

: > > : (I was also under the impression that "centrifugal force" was made up


: > > : to describe phenomena, and never existed)

: > >
: > > Within the last 50 years (since I was in school),


: > > there has been a move by some teachers to eliminate the
: > > term "centrifugal force", which seems a bit restrictive.
: > > What will they call a "centrifuge" if they are
: > > successful? :-)
: > >

: > > : So, (and this sounds like a silly question) who's right: me, or


: > > : Webster's Dictionary <grin>?

: > >
: > > Maybe both are right if the terminology is


: > > straightened out. In the case of the string and the
: > > stone, the acceleration is inward (centripetal), and
: > > a force (inertial) holding the string taunt (centrifugal),
: > > it really isn't a crime to say centrifugal, but it seems
: > > to be out of vogue.
: > >

: WRONG WRONG WRONG ... thrice WRONG
:
: Us 'modern' teachers have it right!

I hope you have it "righter", that would be progress. :-)

: There are NO outward forces holding the string taut, if there were


: then the forces would balance ... there would be NO ACCELERATION.

Then what breaks the string if the RPM is too great?

: Dan Evens has it right ... put another way ...


:
: There is a SINGLE centripetally acting force (caused by pulling the string)

I don't want to consider the case where someone is
pulling on the string, that complicates the problem, and
diverts away from the original question.

: not only on the mass but on every part/element of the string ...

: a single UNBALANCED force causes an acceleration, which is what is
: happening to the string/mass system.

Can we restrict the discussion to a rotating pivot,
a string attached to it, and a stone attached to the other
end of the string?
Then the string does not accelerate.

: The string is NOT in a state of tension in the statics use of the term!

I hope students are not getting more confused by this.

: Remember the whole system is

: ACCELERATING TOWARDS THE CENTRE POINT!

Absolutely not, the string does not accelerate in
any direction, one end is essentially stationary at the
rotating pivot, and the other end stays a constant distance
away.

: X centrifuge X ...

: ... device that contributes lack of centripete to suspended particles!!!!!

I don't understand how a device can contribute a lack of
something.

: There is no case involving circular motion that requires a
: fictitious centrigugal explanation!!!!

If Webster's used it, I am sure the author of that
section was paraphrasing a textbook explanation, it is just
unfortunate the "stone on a string" was mentioned.

: Or do you know better?
: mi...@mikenorr.demon.co.uk
: The English Lake District.

I think I know what inertial forces are, and they
are very real. I would appreciate it if you could quote
directly from the textbook you use, show where it doesn't
agree with Webster's.

Sincerely,

Ken Fischer

Mike Norris

unread,
May 4, 1997, 3:00:00 AM5/4/97
to

In article <E9n5r...@iglou.com>, Ken Fischer
<URL:mailto:kefi...@iglou.com> wrote:

This is getting longer due to misunderstandings!!!!!!

Comments inseted in CAPITALS

> Mike Norris (mi...@mikenorr.demon.co.uk) wrote objections to my
> statements which attempted to explain why Webster's is correct,
> but Mike makes it clear that Webster used a poor example, the
> stone on a string, because for a person to twirl a stone on a
> string, most of the forces are neither centripetal, nor centri-
> fugal, as the hand pulls the string in a circle.
>
> My previous remarks were directed toward a single problem,
> that of a string attached to a rotating pivot, and only considered
> those forces that exist in that case.
>
> I urge all students to follow the teaching of the textbook,
> regardless of what it says. I will reply to Mike at the end
> of this long message, I would like to reduce the size, but it
> isn't possible.
>
> : mi...@mikenorr.demon.co.uk
> : The English Lake District.
> : On Fri 02 May, Mike Norris wrote:
> : > In article <E9HJy...@iglou.com>, Ken Fischer wrote:
> : > >

> : > > I...@Ebon.Psi (ce...@achilles.net) wrote:
> : > > : Webster's 9th New Collegiate Dictionary defines "centrifugal force" on
> : > > : page 221 as the force that an object moving along a circular path
> : > > : exerts on the body constraining the object and that acts outwardly
> : > > : away form the centre of rotation. eg: a stone whirled on a string
> : > > : exerts centrifugal force on the string


WEBSTER IS WRONG .. the person who wrote this does not understand the
nature of circular motion. The stone does not/cannot exert a force.


> : > > : Since when?!?! I was under the impression that the only acceleration


> : > > : on that stone/string problem was centripetal acceleration tangent to
> : > > : the circle in which the stone travels!

YES ... CORRECT


> : > > The only "acceleration" is centripetal, but there
> : > > is such a thing as "centrifugal force" even if it is only
> : > > inertial.

THIS IS THE CRUNCH!!!! you can manufacture/invent artificial/theoretical systems to model
observed behaviour ... but this does NOT mean they are right!!!!!

> : > > : (I was also under the impression that "centrifugal force" was made up


> : > > : to describe phenomena, and never existed)

YES ... CORRECT

> : > > Within the last 50 years (since I was in school),
> : > > there has been a move by some teachers to eliminate the
> : > > term "centrifugal force", which seems a bit restrictive.
> : > > What will they call a "centrifuge" if they are
> : > > successful? :-)
> : > >

> : > > : So, (and this sounds like a silly question) who's right: me, or


> : > > : Webster's Dictionary <grin>?

> : > >
> : > > Maybe both are right if the terminology is
> : > > straightened out. In the case of the string and the
> : > > stone, the acceleration is inward (centripetal), and
> : > > a force (inertial) holding the string taunt (centrifugal),
> : > > it really isn't a crime to say centrifugal, but it seems
> : > > to be out of vogue.

THE STRING IS NOT HELD TAUGHT BY ANY FORCE PULLING OUTWARDS

>
> : WRONG WRONG WRONG ... thrice WRONG
> :
> : Us 'modern' teachers have it right!
>
> I hope you have it "righter", that would be progress. :-)

WHAT?????????????

> : There are NO outward forces holding the string taut, if there were
> : then the forces would balance ... there would be NO ACCELERATION.
>
> Then what breaks the string if the RPM is too great?

YEP OK. IT SHOULD HAVE READ ... NO NET OUTWARD FORCES ...
THE BALANCED INTRA MOLECULAR FORCES HOLDING THE STRING
TOGETHER PLAY NO PART IN THE CENTRIPETAL ACTION ...
THEY ONLY COME INTO PLAY AT THE LIMIT ... LOOK AT A
GRAVITATIONAL ANALOGY TO UNDERSTAND THIS


> : Dan Evens has it right ... put another way ...
> :
> : There is a SINGLE centripetally acting force (caused by pulling the string)
>
> I don't want to consider the case where someone is
> pulling on the string, that complicates the problem, and
> diverts away from the original question.

WRONG ... THIS EXAMPLE IS THE SAME, IN PRINCIPLE, AS ANY OTHER.


> : not only on the mass but on every part/element of the string ...
> : a single UNBALANCED force causes an acceleration, which is what is
> : happening to the string/mass system.
>
> Can we restrict the discussion to a rotating pivot,
> a string attached to it, and a stone attached to the other
> end of the string?
> Then the string does not accelerate.

RUBBISH ... THE STRING IS ALSO ACCELERATING CENTRIPETALLY ...
IS IT NOT ALSO DESCRIBING CIRCULAR MOTION?

> : The string is NOT in a state of tension in the statics use of the term!
>
> I hope students are not getting more confused by this.


IN MY EXPERIENCE STUDENTS ARE NOT CONFUSED AT ALL BY
CENTRIPETAL MOTION ... IT TENDS TO BE OLDER 'STUDENTS' WHO
HAVE ALREADY BEEN CONFUSED BY INCREDIBLY BAD TEACHING
FROM PEOPLE WHO ARE CONFUSED THEMSELVES.

> : Remember the whole system is
> : ACCELERATING TOWARDS THE CENTRE POINT!
>
> Absolutely not, the string does not accelerate in
> any direction, one end is essentially stationary at the
> rotating pivot, and the other end stays a constant distance
> away.

WRONG AGAIN EVERY POINT OFTHE STRING IS ACCELERATING
TOWARDS THE CENTRE ... THE STRING ALSO GOES ROUND IN A CIRCLE


> : X centrifuge X ...
> : ... device that contributes lack of centripete to suspended particles!!!!!
>
> I don't understand how a device can contribute a lack of
> something.

LOOK CAREFULLY AT THE MOTION OF AN INDIVIDUAL PARTICLE
IN THE 'CENRIFUGE' AND ASK WHY IT ISN'T BEING PULLED TO THE
CENTRE OF THE MOTION!


> : There is no case involving circular motion that requires a
> : fictitious centrigugal explanation!!!!
>
> If Webster's used it, I am sure the author of that
> section was paraphrasing a textbook explanation, it is just
> unfortunate the "stone on a string" was mentioned.
>
> : Or do you know better?
> : mi...@mikenorr.demon.co.uk
> : The English Lake District.
>
> I think I know what inertial forces are, and they
> are very real. I would appreciate it if you could quote
> directly from the textbook you use, show where it doesn't
> agree with Webster's.
>
> Sincerely,
>
> Ken Fischer

IT IS CLEAR THAT BY CHANGING FRAMES OF REFERENCE THE CIRCLULAR
MOTION CAN DISAPPEAR AND THE MODEL REQUIRES THE INVENTION
OF AN OUTWARDLY ACTING FORCE.

REMEMBER ITS ONLY A MODEL ... IT IS NEED NOT BE 'CORRECT' IT ONLY
NEEDS TO SHOW WAHT HAPPENS AND PREDICT WHAT MIGHT HAPPEN
IN DIFFERENT CIRCUMSTANCES.

THERE IS ABSOLUTELY NOTHING WRONG WITH THIS APPROACH.

BUT IT ISN'T REAL!!!!!!!!!

THEN WE CAN ASK WHO HAS THE REAL REALITY

I HOPE THIS HELPS

ALSO SINCERELY

Ken Fischer

unread,
May 4, 1997, 3:00:00 AM5/4/97
to

John E. Prussing (prus...@staff.uiuc.edu) wrote:
: >>> In article <E9HJy...@iglou.com>, Ken Fischer
: >>> > Maybe both are right if the terminology is
: >>> > straightened out. In the case of the string and the
: >>> > stone, the acceleration is inward (centripetal), and
: >>> > a force (inertial) holding the string taunt (centrifugal),
: >>> > it really isn't a crime to say centrifugal, but it seems
: >>> > to be out of vogue.
[snip]

: Once again, I submit that a translational, rather than rotational, example


: of an apparent force in a non-inertial frame is easier to understand,
: namely an upwardly accelerating elevator. It illustrates the same concept
: as centrifugal force. An observer in the elevator experiences a downward
: force, let's call it oppceleration, that is opposite in direction to
: the inertial acceleration, just like centrifugal force is opposite in
: direction to centripetal force. The oppceleration is an apparent force
: that is necessary to describe the motion relative to the (non-inertial)
: observer in the frame fixed to the elevator.

All "accelerations" produce "downward" forces, :-)
that is how we determine "up". :-) Some many years ago
I spent some time thinking about a spaceship that could
orbit the Earth in about half the time by going faster
and keeping continuous thrust in an "upward" direction
relative to the Earth's surface.
I think that thrust would have to be about 1 g. :-)

: Back to the rotating frame. I work in orbital mechanics, and the reason


: that an astronaut on the flight deck of the shuttle experiences a "zero-g"
: (a misnomer) or "weightless" (another misnomer) environment is that he
: or she is in circular orbit (this is "circular motion") in a rotating,
: non-inertial frame and the apparent centrifugal force "cancels" the
: gravitational force, resulting in an apparent force-free environment,
: with stuff floating around the flight deck. The orbiting shuttle with
: its on-board tv pictures should be a convincing experiment to prove
: the concept. So what's the problem?

I was hoping that you would say that being in orbit
is simply being _non_ accelerated. :-)

Ken Fischer

Mike Norris

unread,
May 4, 1997, 3:00:00 AM5/4/97
to

In article <E9MyK...@midway.uchicago.edu>,


<URL:mailto:me...@cars3.uchicago.edu> wrote:
>
>
> In article <5kgobg$a...@vixen.cso.uiuc.edu>, prus...@staff.uiuc.edu (John E. Prussing)

> writes:
> >me...@cars3.uchicago.edu writes:

> >>Yes, I do know better.
> >

> >Once again, I submit that a translational, rather than rotational, example
> >of an apparent force in a non-inertial frame is easier to understand,
> >namely an upwardly accelerating elevator. It illustrates the same concept
> >as centrifugal force. An observer in the elevator experiences a downward
> >force, let's call it oppceleration, that is opposite in direction to
> >the inertial acceleration, just like centrifugal force is opposite in
> >direction to centripetal force. The oppceleration is an apparent force
> >that is necessary to describe the motion relative to the (non-inertial)
> >observer in the frame fixed to the elevator.
> >

> Right on.


>
> >Back to the rotating frame. I work in orbital mechanics, and the reason
> >that an astronaut on the flight deck of the shuttle experiences a "zero-g"
> >(a misnomer) or "weightless" (another misnomer) environment is that he
> >or she is in circular orbit (this is "circular motion") in a rotating,
> >non-inertial frame and the apparent centrifugal force "cancels" the
> >gravitational force, resulting in an apparent force-free environment,
> >with stuff floating around the flight deck. The orbiting shuttle with
> >its on-board tv pictures should be a convincing experiment to prove
> >the concept. So what's the problem?
>

> There's no problem. And the astronaut will find it way simpler to
> express the motion of objects on the flight deck using inertial
> forces, than transforming everything to an inertial frame, performing
> the calculations there, then transforming back (in case somebody
> doubts that it is simpler, he's welcome to try).
>
> Mati Meron | "When you argue with a fool,
> me...@cars.uchicago.edu | chances are he is doing just the same"

O.K. all well and good but the point I'm trying to make .. thus far unsuccessfully...
... is that we all use the frame of reference that is suited to the task we are
analysing .. the astronaut being a good example.. but ..
although it may be convenient to model these apparent forces it is not necessary.
We are all familiar with jumping between frames of reference and the
transforms invovled .. but very often these transforms cloud the issue and lead
to confusion .. as is clearly the case with this 'apparent centrifugal force'.
As you have indicated it is not real and all systems can be clearly explained
without it in real terms.

In the everyday world the confusion between real and apparent is rampant
and leads to the muddy thinking evident in some of this discussion!

Sincerely

me...@cars3.uchicago.edu

unread,
May 4, 1997, 3:00:00 AM5/4/97
to

Not necessery as in "you can get all the results without it"? Sure, I
agree, I said so.

>We are all familiar with jumping between frames of reference and the
>transforms invovled .. but very often these transforms cloud the issue and lead
>to confusion .. as is clearly the case with this 'apparent centrifugal force'.
>As you have indicated it is not real and all systems can be clearly explained
>without it in real terms.

They can. But physics is also pragmatic and what often counts a lot
is the ability to get a solution fast, without too much effort, and in
a form which makes the relevant terms stand out clearly. Usually this
means picking a specific reference frame where the results are
especially simple and such frame may often be non inertial. True, the
transformations between various frames may indeed cloud the issue
sometimes but then it is preferable to clarify what's involved rather
then issue a blank statement like "thou shall not use accelerated
frames". I've seen too often students picking an extremely complex
path to solve a problem just due to being conditioned against
"fictitious forces". Now, humans are fallible, as we know, so given
some fixed probability of error per solution step, the more complex
the solution the higher the probability of error.


>
>In the everyday world the confusion between real and apparent is rampant
>and leads to the muddy thinking evident in some of this discussion!

No doubt. And it shows that these issues need to be clarified. But there
is no need to dismiss useful concepts only because they may confuse
some people.

Jim Carr

unread,
May 5, 1997, 3:00:00 AM5/5/97
to

ce...@achilles.net writes:
>
>Webster's 9th New Collegiate Dictionary defines "centrifugal force" on
>page 221 as the force that an object moving along a circular path
>exerts on the body constraining the object and that acts outwardly
>away form the centre of rotation. eg: a stone whirled on a string
>exerts centrifugal force on the string

That is interesting. The ancient and venerable copy of "Webster's
New World Dictionary, College Edition" (1958) that I keep on my
shelf says, and I quote

"_centrifugal_force_, the force tending to make rotating bodies
move away from the center of rotation: it is due to inertia."

Above it reminds of the etymology of cenrtrifugal and that it
was coined by Newton. I would prefer that it say "apparent force"
but the reference to inertia makes this older definition fall in
the category of 'not wrong'. It is certainly accurate for the
time when it was published, since that is the way the term was
used in intro physics at that time if my reading of old books is
at all typical of the genre. However, the old version would only
be accurate (IMO) if it also expanded the comment about inertia
to mention frames -- but they do have limited space.

>Since when?!?! I was under the impression that the only acceleration
>on that stone/string problem was centripetal acceleration tangent to
>the circle in which the stone travels!

That is correct; the centripetal force is what makes the body follow
a curved (accelerated) path. Notice, however, that the 9th edition
version is talking about the force on the *center peg*, not the force
on the body. But I don't like the description they use for it.

>(I was also under the impression that "centrifugal force" was made up
>to describe phenomena, and never existed)

There is a real force exerted by the constraining device (say a
string) on the center. This is the reaction force required to
keep the string in static equilibrium as it exerts a centripetal
force on the body that is being accelerated. That reaction force
is "center fleeing", that is, "centrifugal", because it is equal
and opposite to the centripetal force on the body.

I am not sure I can compose a one-line definition that is complete,
but I would prefer something that stated that the centrifugal force
is the (outward) equal and opposite reaction force *acting on a body*
which is providing the (inward) centripetal force *on an object* that
constrains the object to follow a circular path. I would have the
centripetal force definition mention acceleration rather than make
this sentence any longer.

--
James A. Carr <j...@scri.fsu.edu> | "What are those, Daddy?"
http://www.scri.fsu.edu/~jac/ | Young girl at Smithsonian
Supercomputer Computations Res. Inst. | exhibit of objects and letters
Florida State, Tallahassee FL 32306 | left at the Vietnam Memorial.

Mike Norris

unread,
May 5, 1997, 3:00:00 AM5/5/97
to

In article <5kkpu7$pd1$1...@news.fsu.edu>, Jim Carr
<URL:mailto:j...@ibms48.scri.fsu.edu> wrote:

> There is a real force exerted by the constraining device (say a
> string) on the center. This is the reaction force required to
> keep the string in static equilibrium as it exerts a centripetal
> force on the body that is being accelerated. That reaction force
> is "center fleeing", that is, "centrifugal", because it is equal
> and opposite to the centripetal force on the body.
>
> I am not sure I can compose a one-line definition that is complete,
> but I would prefer something that stated that the centrifugal force
> is the (outward) equal and opposite reaction force *acting on a body*
> which is providing the (inward) centripetal force *on an object* that
> constrains the object to follow a circular path. I would have the
> centripetal force definition mention acceleration rather than make
> this sentence any longer.
>

The reason why you are having such great difficulty in trying to
give a reasoned definition of 'centrifugal force' is that it doesn't exist
as a real force in any reference frame.

The mistake you make is in saying that the string is in static equilibrium...
... it is not ... there is no reaction force! Every part of the string is being
accelerated towards the centre.Every point on the string is executing
circular motion.The force (centripetal) is required to generate this
acceleration. If it were balanced there would be no circular motion ..
the object would fly off at a tangent just like the string being cut,
which prevents the transmision of the centripetal force along the string!

mahipa...@orbital.fsd.com

unread,
May 5, 1997, 3:00:00 AM5/5/97
to

In article <E9oCv...@midway.uchicago.edu>,[trim]

I've already used up my quota of posts per day. Must try break the habit.
But this discussion is really relevant to many of us, students and foe
alike.

The issue of "convenient" vs. "elegant solution" vs. "necessary and
sufficent" has more to it than pragmatism. There are many of us who would
prefer to have the elegant solution methodology to arrive at the equally
elegant solution. From my experience, this usually only leads to Analysis
Paralysis. I'm sure you've experienced the condition.

You sit there performing trajectories of satellites and worry that you
haven't taken into account QM and GR effects. Or somehow the Sphere of
Influence of a particular object was supposed to be greater than you
assumed (i.e., Oprah cheated on her diet this week, for laughter). But
you get the message. There has to be a way to better define the pragmatic
line. If I am not mistaken, one usually turns to the judgement of
experienced individuals for guidance. It's not all that simple. Such
individuals are not exactly freely available. One might do orders of
magnitude comparisons to check for nearness to desired/required accuracy.

Now, is there or isn't there a centrifugal force? It depends on your
level of education or degree of philosophical involvement. Me personally,
I'm too confused and all too happy to not say anything specific. I'm
totally committed to noncommittance. It's more mysterious that way. ;-)
Of course, my personal attributes doesn't resolve your dilemmas. I say
"[HA]!". Enjo(y).

Here's a verse I once wrote long long ago ( =< 1985 ish). Before time,
money, and societal pressures effectively suffocated the romantic in Me:
"Trying to bridge the pieces Between what is real ... Between what I
feel ..."

Of course there's more to it; but I'll be kind and spare you. Still, it
does capture the essence of what many of us as individuals are trying to
accomplish as we study Nature and our part in Her.

One could even make a math out of it:
|what_is_real> - |what_I_feel> = |Error>
where
||Error|| << 0.0000007638 for acceptability.

I know, it still doesn't resolve whether there ought to be or there ought
not to be fictitious forces. It's your dilemma. I've got plenty of
dilemmas of my own for which the choice between pragmatic answers and
elegant solutions is enough to drive me bonkers.

Mahipal |meforce> http://www.geocities.com/Athens/3178/TLoTCoReason.html

-------------------==== Posted via Deja News ====-----------------------
http://www.dejanews.com/ Search, Read, Post to Usenet

Jim Carr

unread,
May 5, 1997, 3:00:00 AM5/5/97
to

mi...@mikenorr.demon.co.uk
:

: There are NO outward forces holding the string taut, if there were
: then the forces would balance ... there would be NO ACCELERATION.

And that is correct, because there is no acceleration of the peg.

Does the stone accelerate? Why?

Does the string accelerate? Why or why not? (identify frame used)

Does the peg holding the string accelerate? Why or why not?

In what way is this problem the same or different from the problem of
a mass hanging over a pulley on the edge of a table, held by a peg?
Could we set this up so the tension in the string and the force on
the peg is identical to the above situation at one instant?

kefi...@iglou.com (Ken Fischer) writes:
>
> Then what breaks the string if the RPM is too great?

The tension, of course, which is what transfers the force from
the peg appying it to the stone being accelerated.

: Dan Evens has it right ... put another way ...
:
: There is a SINGLE centripetally acting force (caused by pulling the string)

Does the string pull on the peg?

Is it correct to describe the force the string applies to the
peg as being center fleeing?

This is just another variant of the old tension problem, always a
sticky one for some students, in 1st year physics.

me...@cars3.uchicago.edu

unread,
May 5, 1997, 3:00:00 AM5/5/97
to

In article <ant05210...@mikenorr.demon.co.uk>, Mike Norris <mi...@mikenorr.demon.co.uk> writes:
>In article <5kkpu7$pd1$1...@news.fsu.edu>, Jim Carr
><URL:mailto:j...@ibms48.scri.fsu.edu> wrote:
>
>> There is a real force exerted by the constraining device (say a
>> string) on the center. This is the reaction force required to
>> keep the string in static equilibrium as it exerts a centripetal
>> force on the body that is being accelerated. That reaction force
>> is "center fleeing", that is, "centrifugal", because it is equal
>> and opposite to the centripetal force on the body.
>>
>> I am not sure I can compose a one-line definition that is complete,
>> but I would prefer something that stated that the centrifugal force
>> is the (outward) equal and opposite reaction force *acting on a body*
>> which is providing the (inward) centripetal force *on an object* that
>> constrains the object to follow a circular path. I would have the
>> centripetal force definition mention acceleration rather than make
>> this sentence any longer.
>> >
> The reason why you are having such great difficulty in trying to
>give a reasoned definition of 'centrifugal force' is that it doesn't exist
>as a real force in any reference frame.

But of course it does. Only, not in an inertial reference frame.

Gregory Loren Hansen

unread,
May 5, 1997, 3:00:00 AM5/5/97
to

In article <ant05210...@mikenorr.demon.co.uk>,
Mike Norris <mi...@mikenorr.demon.co.uk> wrote:

> The reason why you are having such great difficulty in trying to
>give a reasoned definition of 'centrifugal force' is that it doesn't exist
>as a real force in any reference frame.

It's about as real as the force of gravity. Same with coriolis forces.
How about the centrifugal barrier in an orbit problem, which adds an
angular momentum term to the potential energy?

When you're sitting inside an accelerated reference frame, it's often
easiest to use fictitious forces.

--
"Hey, wait a minute. You've got both eyes! You're no special agent.
You're just some jerk who hates my moustache!"
- The Tick

Jim Carr

unread,
May 5, 1997, 3:00:00 AM5/5/97
to

Mike Norris <mi...@mikenorr.demon.co.uk> writes:
>
> The reason why you are having such great difficulty in trying to
>give a reasoned definition of 'centrifugal force' is that it doesn't exist
>as a real force in any reference frame.

Are you saying there is no outward (center fleeing) force on the
peg holding the string? That you do not have to be strong to
execute the Hammer Throw?

>The mistake you make is in saying that the string is in static equilibrium...
>... it is not ... there is no reaction force!

You say that Newton's third law does not apply here?

That there is no equal and opposite force to the centripetal force
being applied to the object in accelerated motion?

>Every part of the string is being accelerated towards the centre.

Equally?

Is there tension in the string?

>If it were balanced there would be no circular motion ..

If it were unbalanced, there would be no need for the person
holding the string. What you must be careful to identify is
what force acts where, and on what body. You are confusing that
detail when you say I have balanced forces on the object.

Notice very carefully that my definition was very careful to say that
the centrifugal force does not act on the object in accelerated motion,
but on the body that is applying the centripetal force to the object.
There is no cancellation of a force on A by a force on B.

>just like the string being cut,
>which prevents the transmision of the centripetal force along the string!

Transmission from where?

Does Newton's third law apply there?

me...@cars3.uchicago.edu

unread,
May 5, 1997, 3:00:00 AM5/5/97
to

In article <8628653...@dejanews.com>, mahipa...@orbital.fsd.com writes:
>In article <E9oCv...@midway.uchicago.edu>,
> me...@cars3.uchicago.edu wrote:
>>
>> >In the everyday world the confusion between real and apparent is rampant
>> >and leads to the muddy thinking evident in some of this discussion!
>>
>> No doubt. And it shows that these issues need to be clarified. But there
>> is no need to dismiss useful concepts only because they may confuse
>> some people.
>
>I've already used up my quota of posts per day. Must try break the habit.
>But this discussion is really relevant to many of us, students and foe
>alike.
>
>The issue of "convenient" vs. "elegant solution" vs. "necessary and
>sufficent" has more to it than pragmatism. There are many of us who would
>prefer to have the elegant solution methodology to arrive at the equally
>elegant solution. From my experience, this usually only leads to Analysis
>Paralysis. I'm sure you've experienced the condition.

Sometimes.


>
>You sit there performing trajectories of satellites and worry that you
>haven't taken into account QM and GR effects. Or somehow the Sphere of
>Influence of a particular object was supposed to be greater than you
>assumed (i.e., Oprah cheated on her diet this week, for laughter). But
>you get the message. There has to be a way to better define the pragmatic
>line.

If there would've been, it would've been an algorithm, not a
"pragmatic line".

If I am not mistaken, one usually turns to the judgement of
>experienced individuals for guidance. It's not all that simple. Such
>individuals are not exactly freely available.

So? Individuals who know how to fix a car aren't freely available
either :-)


>
>Now, is there or isn't there a centrifugal force? It depends on your
>level of education or degree of philosophical involvement.

Nah, you make it sound too profound. Here, lets take an example from
mathematics. Nothing complex, just the quadratic equation

ax^2 + bx + c = 0

with a non zero. Now, we do know that by a proper shift of x we can
eliminate the middle, linear term. So, using the same profound logic
as was exhibited on this thread, we can call this term "the fictitious
term" and threaten with a dire fate any student who dares to write a
quadratic equation with the middle term present. So, is this
sensible? Would you write:

"Now, is there or isn't there a linear term in quadratic equation? It

depends on your >level of education or degree of philosophical

involvement." ?

Ken Fischer

unread,
May 6, 1997, 3:00:00 AM5/6/97
to

I a response to Jim Carr, Mike Norris (mi...@mikenorr.demon.co.uk) wrote:
: The reason why you are having such great difficulty in trying to

: give a reasoned definition of 'centrifugal force' is that it doesn't exist
: as a real force in any reference frame.

Centrifugal "force" is the amount of inertial
resistance that a given body exerts at a given acceleration,
just like the "F" in Newton's formula for gravitation is
the gravitational "force" which is the inertial resistance
a given body exerts in a given gravitational "field", they
are both inertial, and identical in nature and magnitude
for the same body in like accelerations.

: The mistake you make is in saying that the string is in static

: equilibrium... ... it is not ... there is no reaction force!

That is exactly what inertia is, a "reaction" force,
as opposed to a force produced by "work", such as in an
electric motor, or a hydraulic cylinder.

: Every part of the string is being accelerated towards the centre.

Is the pivot point accelerated towards the center?

: Every point on the string is executing circular motion.

Except the pivot point.

: The force (centripetal) is required to generate this
: acceleration.

This is nonsense in that in a friction free
device, the circular motion would continue forever,
so there can be no "force" involved in generating
an "acceleration", the only "force" acting is inertia
trying to prevent all parts of the system from not
moving in a straight line.

: If it were balanced there would be no circular motion ..
: the object would fly off at a tangent just like the string being cut,

: which prevents the transmision of the centripetal force along the string!

: mi...@mikenorr.demon.co.uk
: The English Lake District.

The only thing the string has is tensile strength,
which is a non-work entity. Does a rotating wheel
"accelerate", I think it just goes round and round. :-)

Ken Fischer

mahipa...@orbital.fsd.com

unread,
May 6, 1997, 3:00:00 AM5/6/97
to

In article <E9qBv...@midway.uchicago.edu>,[trim]

> >
> >Now, is there or isn't there a centrifugal force? It depends on your
> >level of education or degree of philosophical involvement.
>
> Nah, you make it sound too profound. Here, lets take an example from
> mathematics. Nothing complex, just the quadratic equation
>
> ax^2 + bx + c = 0
>
> with a non zero. Now, we do know that by a proper shift of x we can
> eliminate the middle, linear term. So, using the same profound logic
> as was exhibited on this thread, we can call this term "the fictitious
> term" and threaten with a dire fate any student who dares to write a
> quadratic equation with the middle term present. So, is this
> sensible? Would you write:
>
> "Now, is there or isn't there a linear term in quadratic equation? It
> depends on your >level of education or degree of philosophical
> involvement." ?

I can't help making stuff sound profound, it's like an occupational
hazard. I just can't wash it off anymore. It's kind of nice actually.
Makes the Fun funnier. Or some such.

The standard form of the parabola representation vs. the quadratic form
representation given above is a great analogy. The translation of the
location of the origin point doesn't alter the fact that each formulation
is equivalently representing the same trajectory==parabola.

[You got to forgive my writing style. I'm practicing writing for journals
now. I can't be as free with my choice of words as I usually am. I sure
hope it's dry and dull enough. ;-) ]

The bottom line is that the choice of reference frame, whether rotating
or translating, whether inertial or non-inertial, with the caveat that a
rotating reference frame could never (by definition) be inertial, leads
to the appearance or absence of "fictitious forces".

Am I right about the caveat? I think yes. I would appreciate being
corrected. I lack reference material to verify the caveat which I can't
refrain myself from making/posting. I talk too much. ;-)

D'Alembert's Principle which essentially takes "F=ma" and rewrites it as
"F-ma=0" is something worth considering in this discussion. In
D'Alembert's formulation, the inertial term "ma" itself becomes a force
and all of dynamics is reduced to a statics problem. The principle is
basically a vector translation. It leads to all sort of confusing
rhetoric built around the words "virtual work". Which, for all intents
and purposes, might as well be "fictitious work". That's the kind of work
I want get paid megabucks for.

All this real confusion really isn't about rhetoric, and the proper
choice of. In treating physics problems, some coordinate frames are
simply easier to work with than others. Furthermore, the choice of
"centrifugal force" is accurate because it leads to practical useful
results. In fact, it gives the ***equivalent results*** had one done the
proper transformation to an actual inertial frame! Same results only
faster. Sounds like a bargain to me.

Now, for experiment, just try spinning one of your kids around by grabing
their hands. Do the free-body-diagrams on both you and your child; then
identify which forces yield the correct result in the least amount of
thinking/time. Pragmatism counts. The forces *you feel* are part of the
set of nonfictitious forces.

Coordinate Transformations and their implications couldn't be stressed
enough. It's fun being disoriented. ;-) It's fun pretending to be
profound too. And if one is profound and disoriented all at once, then
one's had enough martinis.

``All concepts of "force" are made up to describe phenomena.''
John Baez

Mike Norris

unread,
May 6, 1997, 3:00:00 AM5/6/97
to

In article <5klmg6$c0v$1...@news.fsu.edu>, Jim Carr
<URL:mailto:j...@ibms48.scri.fsu.edu> wrote:

>
>
> Mike Norris <mi...@mikenorr.demon.co.uk> writes:
> >
> > The reason why you are having such great difficulty in trying to
> >give a reasoned definition of 'centrifugal force' is that it doesn't exist
> >as a real force in any reference frame.
>
> Are you saying there is no outward (center fleeing) force on the
> peg holding the string? That you do not have to be strong to
> execute the Hammer Throw?

YES ... you got it ... which way does the peg move if the string breaks?!!!
Have you never noticed which way the hammer thrower moves when he/she
releases? .... nowhere!!!

I'm sure a Momentum study would help here.

> >The mistake you make is in saying that the string is in static equilibrium...
> >... it is not ... there is no reaction force!
>

> You say that Newton's third law does not apply here?

No ... 3rd law pair forces act on different bodies ... so that's O.K.

> That there is no equal and opposite force to the centripetal force
> being applied to the object in accelerated motion?

Yes ...

> >Every part of the string is being accelerated towards the centre.
>

> Equally?

Yes

> Is there tension in the string?

Yes .. but it is a confusing word in this situation, as it is linked with
tension pairs in static cases .. use force instead.

> >If it were balanced there would be no circular motion ..
>

> If it were unbalanced, there would be no need for the person
> holding the string. What you must be careful to identify is
> what force acts where, and on what body. You are confusing that
> detail when you say I have balanced forces on the object.

Agreed ... but not confused ... if you are the peg, it is clear that you are
continually pulling the rock orbiting you towards yourself.
... if you are the rock, it is clear that you are continually pulling the peg,
including that which is attached to it, towards yourself.

In both cases the centripetal motion is caused by a single unbalanced force!

> Notice very carefully that my definition was very careful to say that
> the centrifugal force does not act on the object in accelerated motion,

Agreed

> but on the body that is applying the centripetal force to the object.
> There is no cancellation of a force on A by a force on B.
>

So which is accelerating which and in which direction?

> >just like the string being cut,
> >which prevents the transmision of the centripetal force along the string!
>

> Transmission from where?

You know where!

> Does Newton's third law apply there?

Of course it does. But is of no consequence to the motion!

Consider Binary stars .... hmmm


These discussions show why this has been such a thorny problem over the
years. It seems to me that we are all making the fundamental mistake of not
clearly defining a reference frame before embarking on the matter.

After all the only unique frame of reference is that from which you observe ...

Mike Norris

unread,
May 6, 1997, 3:00:00 AM5/6/97
to

Yes ...

Yes

Agreed

You know where!


............ ON ANOTHER MATTER ..........

Where do you guys get all the time from to write so prolifically in the news groups...
... travelling near light speed no doubt!

me...@cars3.uchicago.edu

unread,
May 6, 1997, 3:00:00 AM5/6/97
to
For the "Business Week" that'll do, but for the WWN you may consider
spicing it up a bit :-)

>The bottom line is that the choice of reference frame, whether rotating
>or translating, whether inertial or non-inertial, with the caveat that a
>rotating reference frame could never (by definition) be inertial, leads
>to the appearance or absence of "fictitious forces".
>

Yep.

>Am I right about the caveat? I think yes.

Sure. Though in GR it may be swept under the rug of metric.

>I would appreciate being
>corrected. I lack reference material to verify the caveat which I can't
>refrain myself from making/posting. I talk too much. ;-)

Don't we all.


>
>D'Alembert's Principle which essentially takes "F=ma" and rewrites it as
>"F-ma=0" is something worth considering in this discussion. In
>D'Alembert's formulation, the inertial term "ma" itself becomes a force
>and all of dynamics is reduced to a statics problem. The principle is
>basically a vector translation. It leads to all sort of confusing
>rhetoric built around the words "virtual work". Which, for all intents
>and purposes, might as well be "fictitious work". That's the kind of work
>I want get paid megabucks for.

Would be nice. The only jobs of this kind I've ever seen were paying
"fictitious meagabucks".


>
>All this real confusion really isn't about rhetoric, and the proper
>choice of. In treating physics problems, some coordinate frames are
>simply easier to work with than others. Furthermore, the choice of
>"centrifugal force" is accurate because it leads to practical useful
>results. In fact, it gives the ***equivalent results*** had one done the
>proper transformation to an actual inertial frame! Same results only
>faster. Sounds like a bargain to me.

My feelings exactly.


>
>Now, for experiment, just try spinning one of your kids around by grabing
>their hands. Do the free-body-diagrams on both you and your child; then
>identify which forces yield the correct result in the least amount of
>thinking/time. Pragmatism counts. The forces *you feel* are part of the
>set of nonfictitious forces.
>

I could run this experiment years ago, but by now my kids are too big.
So I won't even mention it to them, else they get the idea to run it
in reverse :-)

>Coordinate Transformations and their implications couldn't be stressed
>enough. It's fun being disoriented. ;-) It's fun pretending to be
>profound too. And if one is profound and disoriented all at once, then
>one's had enough martinis.

Just enough. One more and all that's left is the "disoriented" part.


>
>``All concepts of "force" are made up to describe phenomena.''
> John Baez

Yeah, I've seen this one and I like it.

Jim Carr

unread,
May 6, 1997, 3:00:00 AM5/6/97
to

Jim Carr <j...@ibms48.scri.fsu.edu> wrote:
|
| Are you saying there is no outward (center fleeing) force on the
| peg holding the string? That you do not have to be strong to
| execute the Hammer Throw?

Mike Norris <mi...@mikenorr.demon.co.uk> writes:
>
>YES ... you got it ... which way does the peg move if the string breaks?!!!

The question is, can the peg break. The answer is, yes. Does a
strain gauge on the peg measure a force? Yes.

In any case, your answer above is inconsistent with your answer
below where you say there is a tension in the string. How is
that tension maintained if the peg does not apply a force to
the string?

>Have you never noticed which way the hammer thrower moves when he/she
>releases? .... nowhere!!!

Not so. Sometimes they fall out of the circle.

| You say that Newton's third law does not apply here?

>No ... 3rd law pair forces act on different bodies ... so that's O.K.

That is at least consistent with your assertion that there is no force
acting on the peg, but then (1) why do you think the peg and object are
the same body and (2) why did you object to the definition, which clearly
said the centrifugal force acts on the peg and the centripetal force
acts on the string/object? I think you did not read the definition
very carefully.

| >Every part of the string is being accelerated towards the centre.
|
| Equally?

>Yes

| Is there tension in the string?

>Yes .. but it is a confusing word in this situation, as it is linked with
>tension pairs in static cases .. use force instead.

No, it is not confusing. The tension is required to provide the
acceleration of the string and tells you that a centripetal force
being applied by the peg to the string -- and thus, by Newton's
third law, that a centrifugal force is being applied to the peg
by the string.

Note well that the definition we are talking about here is not the
1958 version (centrifugal force as the fictitious force arising
from inertia when you are in a non-inertial frame) but the modern
version being objected to (the force on the peg that is equal and
opposite to the force applied by the peg).

| If it were unbalanced, there would be no need for the person
| holding the string. What you must be careful to identify is
| what force acts where, and on what body. You are confusing that
| detail when you say I have balanced forces on the object.

>Agreed ... but not confused ... if you are the peg, it is clear that you are
>continually pulling the rock orbiting you towards yourself.

This contradicts your answer at the top of this article, so you clearly
must be confused. Newton's 3rd law definitely applies here. There is
a force on the peg applied by the string, a force that we can measure
with a strain gauge attached to the peg.

In answer to a related question about the 3rd law, you agree:

>Of course it does. But is of no consequence to the motion!

Of course, but if you read my definition I did not claim that the
centrifugal force on the peg caused anything, only that it is the
reaction force on the peg required when the peg produces the
centripetal force that *does* produce the motion.

> ... if you are the rock, it is clear that you are continually pulling the peg,
>including that which is attached to it, towards yourself.

Now you are in a non-inertial frame invoking a fictitious force.
Interesting.

John E. Prussing

unread,
May 7, 1997, 3:00:00 AM5/7/97
to

Mike Norris <mi...@mikenorr.demon.co.uk> writes:

[deleted]

>O.K. all well and good but the point I'm trying to make .. thus far unsuccessfully...
>... is that we all use the frame of reference that is suited to the task we are
>analysing .. the astronaut being a good example.. but ..
>although it may be convenient to model these apparent forces it is not necessary.

Not necessary, but sometimes it's convenient to describe things relative
to a rotating frame, e.g., sunrise and sunset. Because we live in a
frame rotating with the earth, it's natural to describe things relative
to our frame of reference.

The reference inertial frame in this case is the distant stars.

The *apparent* Coriolis force explains why the air circulation
around a low pressure center is counter-clockwise (in the
northern hemisphere) and clockwise around a high pressure center.
Meteorologists and weather forecasters use this concept all the
time because we are all doomed to live in this wretched rotating
frame, whether we like it or not.

Surely Shakespeare must have written something about this.
Or maybe later, existential writers. If not, somebody should.

John E. Prussing

unread,
May 7, 1997, 3:00:00 AM5/7/97
to

she...@tcg.anl.gov (Ron Shepard) writes:

>E. Prussing) wrote:

>[...]


>> Back to the rotating frame. I work in orbital mechanics, and the reason
>> that an astronaut on the flight deck of the shuttle experiences a "zero-g"
>> (a misnomer) or "weightless" (another misnomer) environment is that he
>> or she is in circular orbit (this is "circular motion") in a rotating,
>> non-inertial frame and the apparent centrifugal force "cancels" the
>> gravitational force, resulting in an apparent force-free environment,
>> with stuff floating around the flight deck. The orbiting shuttle with
>> its on-board tv pictures should be a convincing experiment to prove
>> the concept. So what's the problem?

>If the shuttle were in free fall going straight towards the earth surface,


>then things inside the shuttle would float around too.

This is true, but not good for the astronauts. But you're right,
it's like a falling elevator.

>In fact, the
>shuttle in a stationary orbit (which is an elipse, not just a circle) is a
>special case of free fall.

True also, but it's actually a more general case of free fall than
the rectilinear, straight toward the earth, case of the elevator.

>I'm not exactly sure what is the "problem",
>but it is inconsistent to describe one free-fall situation in a cartesian
>frame with no "extra" forces, but the other one in a polar frame with this
>extra "centrifugal" force. Yes, this is perhaps the best way to describe
>the two cases, but the two cases are really two instances of the same
>free-fall situation.

Yes they are both free-fall, but one has circumferential
motion and the other has only radial motion, i.e., it's a special
case.

>BTW, it seems to me that the centrifugal force
>arises from the use of a polar coordinate system, not from a rotating
>reference frame.

The polar coordinate system IS, by definition, a rotating frame.
The r direction is rotating, r_dot is the velocity component along
a rotating axis direction, r_dotdot is the component of acceleration
along a rotating (non-inertial) axis direction, etc.

>Lots of SR and GR stuff seems to depend on moving and
>rotating reference frames, but centrifugal force is not one of them.

One of what?

John E. Prussing

unread,
May 7, 1997, 3:00:00 AM5/7/97
to

kefi...@iglou.com (Ken Fischer) writes:

>John E. Prussing (prus...@staff.uiuc.edu) wrote:
>: >>> In article <E9HJy...@iglou.com>, Ken Fischer
>: >>> > Maybe both are right if the terminology is
>: >>> > straightened out. In the case of the string and the
>: >>> > stone, the acceleration is inward (centripetal), and
>: >>> > a force (inertial) holding the string taunt (centrifugal),
>: >>> > it really isn't a crime to say centrifugal, but it seems
>: >>> > to be out of vogue.
>[snip]

>: Once again, I submit that a translational, rather than rotational, example


>: of an apparent force in a non-inertial frame is easier to understand,
>: namely an upwardly accelerating elevator. It illustrates the same concept
>: as centrifugal force. An observer in the elevator experiences a downward
>: force, let's call it oppceleration, that is opposite in direction to
>: the inertial acceleration, just like centrifugal force is opposite in
>: direction to centripetal force. The oppceleration is an apparent force
>: that is necessary to describe the motion relative to the (non-inertial)
>: observer in the frame fixed to the elevator.

> All "accelerations" produce "downward" forces, :-)


>that is how we determine "up". :-)

>Some many years ago
>I spent some time thinking about a spaceship that could
>orbit the Earth in about half the time by going faster
>and keeping continuous thrust in an "upward" direction
>relative to the Earth's surface.
>I think that thrust would have to be about 1 g. :-)

I think you mean that the thrust would act downward, increasing effective
value of the gravitational acceleration of the earth. This would require
a greater velocity to generate the requisite centrifugal force to cancel
it and remain in orbit!

The thrust would actually be directed down, but the rocket engine exhaust
would be up as you imply. But the propellant requirements are enormous
for a sustained orbit. Who's going to pay for this?

>: Back to the rotating frame. I work in orbital mechanics, and the reason


>: that an astronaut on the flight deck of the shuttle experiences a "zero-g"
>: (a misnomer) or "weightless" (another misnomer) environment is that he
>: or she is in circular orbit (this is "circular motion") in a rotating,
>: non-inertial frame and the apparent centrifugal force "cancels" the
>: gravitational force, resulting in an apparent force-free environment,
>: with stuff floating around the flight deck. The orbiting shuttle with
>: its on-board tv pictures should be a convincing experiment to prove
>: the concept. So what's the problem?

> I was hoping that you would say that being in orbit


>is simply being _non_ accelerated. :-)

True. One can be contentedly at rest in a rotating frame and be
(inertially) accelerating like crazy, but blissfully unaware of it.
Isn't life wonderful?

>Ken Fischer

mahipa...@orbital.fsd.com

unread,
May 8, 1997, 3:00:00 AM5/8/97
to

In article <E9rzu...@midway.uchicago.edu>,
me...@cars3.uchicago.edu wrote:

>
> In article <8629285...@dejanews.com>, mahipa...@orbital.fsd.com writes:
[trim]

> >Now, for experiment, just try spinning one of your kids around by grabing


> >their hands. Do the free-body-diagrams on both you and your child; then
> >identify which forces yield the correct result in the least amount of
> >thinking/time. Pragmatism counts. The forces *you feel* are part of the
> >set of nonfictitious forces.
> >
> I could run this experiment years ago, but by now my kids are too big.
> So I won't even mention it to them, else they get the idea to run it
> in reverse :-)

You mean like _COUNTER_ clockwise?? ;-)

Holy Despin Batman! And if you despin and detumble at just the right
resonance frequency you could begin to get younger once again? Mati,
don't let the secret out before you got all the legal paperwork and
patent firmly in place. Shhhh. Keep very very hush.

> >Coordinate Transformations and their implications couldn't be stressed
> >enough. It's fun being disoriented. ;-) It's fun pretending to be
> >profound too. And if one is profound and disoriented all at once, then
> >one's had enough martinis.
>
> Just enough. One more and all that's left is the "disoriented" part.

You are one very sensible physicist/guy. You know the concept of
Thresholds, in varied subjects, very very well.

me...@cars3.uchicago.edu

unread,
May 8, 1997, 3:00:00 AM5/8/97
to

In article <8631062...@dejanews.com>, mahipa...@orbital.fsd.com writes:
>In article <E9rzu...@midway.uchicago.edu>,
> me...@cars3.uchicago.edu wrote:
>>
>> In article <8629285...@dejanews.com>, mahipa...@orbital.fsd.com writes:
>[trim]
>
>> >Now, for experiment, just try spinning one of your kids around by grabing
>> >their hands. Do the free-body-diagrams on both you and your child; then
>> >identify which forces yield the correct result in the least amount of
>> >thinking/time. Pragmatism counts. The forces *you feel* are part of the
>> >set of nonfictitious forces.
>> >
>> I could run this experiment years ago, but by now my kids are too big.
>> So I won't even mention it to them, else they get the idea to run it
>> in reverse :-)
>
>You mean like _COUNTER_ clockwise?? ;-)
>
>Holy Despin Batman! And if you despin and detumble at just the right
>resonance frequency you could begin to get younger once again?

Hey, that's an interesting angle :-)

>Mati, don't let the secret out before you got all the legal paperwork
>and patent firmly in place. Shhhh. Keep very very hush.

But of course. It is all Top Secret. Delete before reading.

mahipa...@orbital.fsd.com

unread,
May 9, 1997, 3:00:00 AM5/9/97
to

In article <5komt3$d...@vixen.cso.uiuc.edu>,

prus...@staff.uiuc.edu (John E. Prussing) wrote:
>
> she...@tcg.anl.gov (Ron Shepard) writes:
[trim]

> >BTW, it seems to me that the centrifugal force
> >arises from the use of a polar coordinate system, not from a rotating
> >reference frame.
>
> The polar coordinate system IS, by definition, a rotating frame.
> The r direction is rotating, r_dot is the velocity component along
> a rotating axis direction, r_dotdot is the component of acceleration
> along a rotating (non-inertial) axis direction, etc.

You are both misleading eachother and maybe others. The choice of polar
coordinates over rectangular coordinates is absolutely not the reason for
the appearance of "centrifugal" force. The keyword is ROTATION, as in
moving in a circular fashion.

Coordinate transformations are arbitrary and individuals use specific
ones to solve specific problems because of geometric symmetries and
simplicities. For example, polar coordinates are "better" for circular
motion and rectangular ones are "better" for linear motion. The choice of
the *geometry* of the coordinate frame does NOT in any way _cause_ the
presence or absence of fictitious forces. That phenomena is a consequence
of the ROTATION. A polar frame is not moving any more than a cartesian
frame. The ROTATION is really the key factor.

As a matter of fact, all problems, however complicated, can be
equivalently represented and analyzed in any available choice of
geometric reference frame. It just so happens, that Cartesianism is the
easiest and simplest one to work with for most people. The choice of a
Cartesian frame over a Spherical Frame or WhatEverHaveYou Frame doesn't
change the dynamics (== forces) present in a given situation. As such,
and this is a little known insiders secret, if you have a new problem to
solve, always start with simplest coordinate basis to you. Then,
afterwards, you can transform to more complicated ones and both impress
and intimdate the geometrically challenged. Just don't tell 'em how you
got to the complicated stuff. Be hush. Happy orienting.

In sum, the dynamical analysis of the twirling-your-kid problem doesn't
change because of the choice of polar frames over rectangular ones. The
centrifugal force is present in both of them. Each frame is as
non-inertial as the other one. Believe me, I've tried to find a true
inertial frame. No luck at all.

John E. Prussing

unread,
May 12, 1997, 3:00:00 AM5/12/97
to

mahipa...@orbital.fsd.com writes:

>In article <5komt3$d...@vixen.cso.uiuc.edu>,
> prus...@staff.uiuc.edu (John E. Prussing) wrote:
>>
>> she...@tcg.anl.gov (Ron Shepard) writes:
>[trim]
>> >BTW, it seems to me that the centrifugal force
>> >arises from the use of a polar coordinate system, not from a rotating
>> >reference frame.
>>
>> The polar coordinate system IS, by definition, a rotating frame.
>> The r direction is rotating, r_dot is the velocity component along
>> a rotating axis direction, r_dotdot is the component of acceleration
>> along a rotating (non-inertial) axis direction, etc.

>You are both misleading eachother and maybe others. The choice of polar
>coordinates over rectangular coordinates is absolutely not the reason for
>the appearance of "centrifugal" force. The keyword is ROTATION, as in
>moving in a circular fashion.

Is "absolutely" a pun here?

I think you missed my point. In orbital motion the polar coordinate frame
is inherently rotating with respect to an inertial frame. That's why
m * r dotdot does not equal the radial component of force, i.e., Newton's
2nd Law does not apply. The absolute acceleration contains centrifugal,
coriolis terms, etc.

Of course one could also define a rotating cartesian frame if one wanted
to. But what's the point? A rotating x-axis would be just like the
polar r-axis. In a nice, safe, inertial cartesian frame m * x dotdot
= the x-component of the applied force, etc.

Of course rotation is the key factor, that's pretty obvious.
That was my point. Rotation is one of two culprits that make a frame
non-inertial. It's left as an exercise to the Gentle Reader to state
what the other culprit is. But please be as brief as possible.

mahipa...@orbital.fsd.com

unread,
May 12, 1997, 3:00:00 AM5/12/97
to

In article <5l5p39$b...@vixen.cso.uiuc.edu>,

prus...@staff.uiuc.edu (John E. Prussing) wrote:
>
> mahipa...@orbital.fsd.com writes:
>
> >In article <5komt3$d...@vixen.cso.uiuc.edu>,
> > prus...@staff.uiuc.edu (John E. Prussing) wrote:
> >>
> >> she...@tcg.anl.gov (Ron Shepard) writes:
> >[trim]
> >> >BTW, it seems to me that the centrifugal force
> >> >arises from the use of a polar coordinate system, not from a rotating
> >> >reference frame.

Read Ron Shepard's line above. And keep it in perpective.

> >> The polar coordinate system IS, by definition, a rotating frame.
> >> The r direction is rotating, r_dot is the velocity component along
> >> a rotating axis direction, r_dotdot is the component of acceleration
> >> along a rotating (non-inertial) axis direction, etc.
>
> >You are both misleading eachother and maybe others. The choice of polar
> >coordinates over rectangular coordinates is absolutely not the reason for
> >the appearance of "centrifugal" force. The keyword is ROTATION, as in
> >moving in a circular fashion.
>
> Is "absolutely" a pun here?

Not really a pun. Sometimes the use of "absolute" is absolutely
legitimate. One, including myself, has to be very careful when using the
word/concept. "Everything is relative" doesn't permeate and transcend
everything.

> I think you missed my point. In orbital motion the polar coordinate frame
> is inherently rotating with respect to an inertial frame. That's why
> m * r dotdot does not equal the radial component of force, i.e., Newton's
> 2nd Law does not apply. The absolute acceleration contains centrifugal,
> coriolis terms, etc.

Be very careful about saying F=dp/dt "does not apply". Go back and review
how the centrifugal and coriolis terms arise out of v=v_0 + w x r in the
first place.

> Of course one could also define a rotating cartesian frame if one wanted
> to. But what's the point? A rotating x-axis would be just like the
> polar r-axis. In a nice, safe, inertial cartesian frame m * x dotdot
> = the x-component of the applied force, etc.

My point is that the choice of the geometry (cartesian, polar, spherical,
bipolar, and at least 8 others) is not the issue of inertial frames.
Physicists don't sit around pondering whether the Universe is ultimately
rectangular or spherical or bipolar. The choice of geometric reference
frames to represent motions of particles is quite arbitrary. The choice
of polar coordinates is not the reason for the presence of a centrifugal
force term. In all coordinate basis, the centrifugal force term exists
for non-inertial frames.

> Of course rotation is the key factor, that's pretty obvious.
> That was my point. Rotation is one of two culprits that make a frame
> non-inertial. It's left as an exercise to the Gentle Reader to state
> what the other culprit is. But please be as brief as possible.

Agreed. The _only_ culprit is acceleration. An inertial frame can not be
accelerating. It's the acceleration of a rotating frame which makes it
non-inertial. There's really only one culprit. [Correct me if I err,
please.]

Another way to define an inertial frame is to state "That frame in which
Newton's F=ma applies is an inertial frame." But Einstein's proved Newton
didn't know physics all that well. Touche. So if Newton could be wrong,
then what's the big deal of some us Netizens seem disoriented wrt physics
knowledge? :-|

Because of the 4D Nature of Spacetime, it kind of seems obtuse to try
find a 3D spatial inertial frame anyway. If it's of any comfort, *if*
there is a inertial frame in Existence, it's oriented wrt you in some
direction at all times. Want the quaternion transformations to get to it?
;-)

Hamilton, Robert Bryan

unread,
May 12, 1997, 3:00:00 AM5/12/97
to

I hope I got the attributions right...

On 12 May 1997 00:43:21 GMT, John E. Prussing <prus...@staff.uiuc.edu> wrote:
>>> The polar coordinate system IS, by definition, a rotating frame.

Nope, not by definition.

>......... In orbital motion the polar coordinate frame


>is inherently rotating with respect to an inertial frame. That's why
>m * r dotdot does not equal the radial component of force, i.e., Newton's
>2nd Law does not apply. The absolute acceleration contains centrifugal,
>coriolis terms, etc.

This doesnt ring true at all. the quantity m r dotdot, where r is a vector,
is the accelleration in either polar or rectangular coordinates. The only
difference is the way we express the vector. Newton's
second law applies just fine, without any centrifugal, etc terms.

The polar coordinate system definitely does not have to be rotating.
Now you can of course use a rotating system if you so desire, but
that's a different matter.

me...@cars3.uchicago.edu

unread,
May 12, 1997, 3:00:00 AM5/12/97
to

In article <8634685...@dejanews.com>, mahipa...@orbital.fsd.com writes:
>In article <5l5p39$b...@vixen.cso.uiuc.edu>,

>Another way to define an inertial frame is to state "That frame in which
>Newton's F=ma applies is an inertial frame."

No, even this doesn't cut it. F = ma applies perfectly well in non
inertial frames, provided you use inertial forces. What you should've
said is "a frame in which F = ma applies, where F doesn't include
contributions which can be globally eliminated by a frame
transformation.

>But Einstein's proved Newton didn't know physics all that well.

Now, where did this bit of childishness come from? Yet another version
of history?

Peter Diehr

unread,
May 13, 1997, 3:00:00 AM5/13/97
to

me...@cars3.uchicago.edu wrote in article ...
>>But Einstein's proved Newton didn't know physics all that well.
>
>Now, where did this bit of childishness come from? Yet another version
>of history?
>

You are correct to question this, Mati! Einstein praised Newton many
times. In one of his brief essays, he says something like "Newton
accomplished as much as was possible for a first class mind, given the
developments of the age".

I see no criticism of Newton in this.

Best Regards, Peter

Peter Diehr

unread,
May 13, 1997, 3:00:00 AM5/13/97
to

Matt McIrvin

unread,
May 13, 1997, 3:00:00 AM5/13/97
to

> No, even this doesn't cut it. F = ma applies perfectly well in non
> inertial frames, provided you use inertial forces. What you should've
> said is "a frame in which F = ma applies, where F doesn't include
> contributions which can be globally eliminated by a frame
> transformation.

Is even this satisfactory? It seems to me that a perverse physicist could
argue that the force in a nonrotating frame is the force in some rotating
frame plus *minus* the centrifugal and Coriolis forces. When I transform
to the rotating frame, I "eliminate" that component, leaving only the
force in the rotating frame...

(I'm just trying to cause trouble.)

--
Font-o-Meter! Proportional Monospaced
^
http://world.std.com/~mmcirvin/

me...@cars3.uchicago.edu

unread,
May 13, 1997, 3:00:00 AM5/13/97
to
Neither do I. But it seems that we're doomed to struggle, eternally,
against the naive view of the history of science as a process where
every generation somebody proves that all his predecessors were just
stupid. It is but a half year since the infamous "Newton was wrong"
thread and here we go again.

mahipa...@orbital.fsd.com

unread,
May 14, 1997, 3:00:00 AM5/14/97
to
> In article <8634685...@dejanews.com>, mahipa...@orbital.fsd.com
writes:
> >In article <5l5p39$b...@vixen.cso.uiuc.edu>,
>
> >Another way to define an inertial frame is to state "That frame in which
> >Newton's F=ma applies is an inertial frame."
>
> No, even this doesn't cut it. F = ma applies perfectly well in non
> inertial frames, provided you use inertial forces. What you should've
> said is "a frame in which F = ma applies, where F doesn't include
> contributions which can be globally eliminated by a frame
> transformation.

I wrote that from memory of what I once heard. Really, I didn't question
its validity. I should not have used the word "define". Actually, that
statement is nearly verbatim what a dynamics (graduate level) professor
said convincingly in a class of 40 students, I think I was #39. Way at
the end of the class! It was the first day of class, so I think a lot of
student were sampling courses and professors to decide what to sign up
for. 40 is too high for graduate classes.

But F=ma is *said* to hold exactly in inertial frames. But I appreciate
the correction since I should not have written the statement so
definitively. I got careless. Yet, I would be surprised if many many
students haven't heard the very same statement. Consider:

Student: How do you define an inertial frame?
Teacher: An inertial frame is one in which F=ma holds.

What's needed to correct this exchange between student&teacher?

> >But Einstein's proved Newton didn't know physics all that well.
>
> Now, where did this bit of childishness come from? Yet another version
> of history?

I was just having fun! It was kind of childish also. Not to mention, I
wrote that with considerable sadness and hesitation. But it had to be
said. Regardless of how I personally feel about Newton's work, there is a
*popular* media propelled image that Newton really missed the boat
because of later developments in both QM and GR. A specific example would
be the precession of Mercury blunder. Well, allow me my rhetoric. I trust
I have good intentions regards physics. I think I've sufficently spent
adequate time, and sincerely so, defending Newton's work against "Newton
is wrong" sloganeering. Yet, one has to be realistic. One has to know
from where these slogans originate.

Q: Who was experimentally more correct, Einstein or Newton wrt Mercury's
data? A: Einstein. By Occam's Razor, we must accept the consequent
implications.

As many are well aware, "forces" really aren't anymore since we now have
"geodesics". It's an either/or dilemma for most. You, I, and the more
educated ones recognize the crucial role that Newton played in setting
the stage for GR to be realized. We can solve problems using either
method. [I need some practice with GR, but I think I'll have no problem
really]. Yet, non-practitioners of physics but true-to-concepts-followers
want the "best" answer. Either particles follow geodesics or there are
forces. In fact, it's the PhD physicists who would prefer people think in
geodesics. They perpetuate the dilemma. As a matter of experienced fact,
one could be belittled in certain circles for using the outdated
"forces". Not me, of course. I too clever.

Mati, it was easier being childish. ;-)

Jim Carr

unread,
May 14, 1997, 3:00:00 AM5/14/97
to

me...@cars3.uchicago.edu wrote:

}
} mahipa...@orbital.fsd.com writes:
} >
} >Another way to define an inertial frame is to state "That frame in which
} >Newton's F=ma applies is an inertial frame."
}
} No, even this doesn't cut it. F = ma applies perfectly well in non
} inertial frames, provided you use inertial forces. What you should've
} said is "a frame in which F = ma applies, where F doesn't include
} contributions which can be globally eliminated by a frame
} transformation.

mahipa...@orbital.fsd.com writes:
>
>I wrote that from memory of what I once heard.

Or maybe you just read it in the Principia.

But you do have to have defined "force" first as Meron points out;
at least that is what Newton did.

me...@cars3.uchicago.edu

unread,
May 14, 1997, 3:00:00 AM5/14/97
to
>But F=ma is *said* to hold exactly in inertial frames.

In any frame, if you include inertial forces.

>But I appreciate
>the correction since I should not have written the statement so
>definitively. I got careless. Yet, I would be surprised if many many
>students haven't heard the very same statement. Consider:
>
>Student: How do you define an inertial frame?
>Teacher: An inertial frame is one in which F=ma holds.
>
>What's needed to correct this exchange between student&teacher?

There were quite a few posts on this subject during the last two days,
so I guess this is covered.


>
>> >But Einstein's proved Newton didn't know physics all that well.
>>
>> Now, where did this bit of childishness come from? Yet another version
>> of history?
>
>I was just having fun! It was kind of childish also. Not to mention, I
>wrote that with considerable sadness and hesitation. But it had to be
>said. Regardless of how I personally feel about Newton's work, there is a
>*popular* media propelled image that Newton really missed the boat
>because of later developments in both QM and GR.

I know. That's why I did feel obliged to comment since, like most
stuff dissiminated by the popular media, this is pure bullshit.

>A specific example would be the precession of Mercury blunder.

Oh, yeah. The "huge" precession, a whooping 10^(-8) relative change.
How could they miss it?-) "Guess what, Ma, we've been lied to all
these years. Turns out that Mt. Everest is half a milimeter lower
than what the books say". Took another two hundred years after
Newton before it could've even be measured.


>
>Q: Who was experimentally more correct, Einstein or Newton wrt Mercury's
>data?

How the hell can somebody be correct or incorrect wrt data that
doesn't exist at the time he does his work?


>
>As many are well aware, "forces" really aren't anymore since we now have
>"geodesics".

Really? Describe EM forces in terms of geodesics, please.


>
>Mati, it was easier being childish. ;-)
>

Oh, I know. That's why it is so popular :-)

Ron Shepard

unread,
May 14, 1997, 3:00:00 AM5/14/97
to

In article <5lb6ii$o...@vixen.cso.uiuc.edu>, prus...@staff.uiuc.edu (John
E. Prussing) wrote:

[...]


>>> >> she...@tcg.anl.gov (Ron Shepard) writes:
>>> >[trim]
>>> >> >BTW, it seems to me that the centrifugal force
>>> >> >arises from the use of a polar coordinate system, not from a rotating
>>> >> >reference frame.
>

> [...] As a
>practical matter, for spacecraft traveling in our solar system, a
>perfectly good inertial frame over a decade or so is one centered in
>our sun and non-rotating with repsect to distant stars. [...]

This is what I meant when I said that it is the polar coordinate system,
not the rotating reference frame that gives rise to "centrifugal force".
This is the _standard_ treatement in fact. For example, see Marion's
"Classical Dynamics" pp. 247-278 or so. The L^2/(2*mu*r^2) term is added
to the potential energy, and the derivative of this w.r.t. "r" is called
the "centrifugal force". Forces are often defined as derivatives of
potential energy expressions, but in this case it is convenient to include
this piece of the kinetic energy since L is a conserved quantity. Whether
the "theta" part of the coordinate system is inertially fixed or rotating
doesn't really enter the argument.

$.02 -Ron Shepard

Xtremethug

unread,
May 15, 1997, 3:00:00 AM5/15/97
to

My teacher put it this way:

Centrifugal force is ficticious. What is pushing the "object"
outwards is the law of inertia (straight line motion unless acted upon by
an outside force) and of course that outside force is centrepital force

mahipa...@orbital.fsd.com

unread,
May 15, 1997, 3:00:00 AM5/15/97
to

In article <EA564...@midway.uchicago.edu>,

me...@cars3.uchicago.edu wrote:
>
> In article <5lamfi$2...@news2.ic.net>, "Peter Diehr" <pdi...@mail.ic.net>
writes:
> > me...@cars3.uchicago.edu wrote in article ...
> >>>In article <5l5p39$b...@vixen.cso.uiuc.edu>,

> >>>But Einstein's proved Newton didn't know physics all that well.
> >>
> >>Now, where did this bit of childishness come from? Yet another version
> >>of history?
> >>
> >
> >You are correct to question this, Mati! Einstein praised Newton many
> >times. In one of his brief essays, he says something like "Newton
> >accomplished as much as was possible for a first class mind, given the
> >developments of the age".
> >
> >I see no criticism of Newton in this.
> >
> Neither do I. But it seems that we're doomed to struggle, eternally,
> against the naive view of the history of science as a process where
> every generation somebody proves that all his predecessors were just
> stupid. It is but a half year since the infamous "Newton was wrong"
> thread and here we go again.

Both of you need to get a perspective. Calm down. You write as if I said
Einstein made personal comments about Newton. The one statement of mine
above is just: "Einstein's work proved that Newton ..."

It's common practice to contract sentences like that once in a while.

Sorry if the statement bothers you. But you need remember that due to
natural advances in experimental sciences, it is more likely than not
that future physicists will say *we* *TODAY* also didn't know physics
very well. Of course, all simply because future physicists will have the
advantage of hindsight. There's no defeat nor victory in this.

Just for the record, I resent being relegated to the crowds that are
implied by the "Newton was wrong" thread. I realize that was a traumatic
four months for Usenet, but hell, I was one of those who insisted "Newton
was absolutely right." So get a fix before you misinterpret and
misrepresent and essentially malign someone. Even by the implication.

You've gone back and forth with three posts and my one line. The line is
even taken out of context. The single sentence word immediately following
and connected to the line above was is "Touche.". That's supposed to mean
something quite ordinary in common usage all over town. It reflects the
author's involuntary and sad acceptance that there is some painful truth
in the message.

I'm kind of surprised by this exchange of yours. But I suspect you feel
better now that you've exorcised the ghosts and demons that haunt your
readings of peoples' writing. If you want to attack someone that actually
holds and perpetuates those views, get YOUR AIM RIGHT! Try popular media,
physics departments, and experimental evidence. Try mathematics also.

Hecht, historically Newton wasn't all that nice a guy either. Apparently
he screwed both Hooke and Leibniz. I can only imagine what treatment
Einstein would've gotten from Newton had Einstein discovered GR and had
to present it to the Royal Academy of which Newton was head hancho. This
fictional setting does assume both individuals as contemporaries.
[WARNING: This is a hypothetical setting. Read with extreme caution
before you exercise your extreme prejudice! WARNING. DANGER.]

Not that Newton's or anyone's personal actions and attitudes are of any
consequence to F=ma or G_{un}=T_{un} or H|v>=E|v> or ...

Ken Fischer

unread,
May 15, 1997, 3:00:00 AM5/15/97
to

Xtremethug (xtrem...@aol.com) wrote:
: My teacher put it this way:

: Centrifugal force is ficticious. What is pushing the "object"
: outwards is the law of inertia (straight line motion unless acted upon by
: an outside force) and of course that outside force is centrepital force

That is correct, no physicist will argue with that.
But inertia is real, and perhaps one of the most powerful forces
in the universe.
The arguement has been about the use of the word
"centrifugal" force, some people seem to feel the word
should not be used. I see nothing wrong with using
the word if the user knows what it really is.
The case of the string and stone is usually used
when discussing centripetal force, and some even go so
far as to say centripetal "acceleration", which in some
cases is ok, but in other cases may be misleading also.

A train moving on a circular track is forced to
depart from inertial motion by the track, but the track
does not accelerate.
An automobile going around a curve is forced to
depart from inertial motion, and the road does not
accelerate.
The use of one word or the other should be
acceptable to anyone, provided they understand that
"centrifugal force" is inertia at work.
And that "centripetal" _acceleration_ is a
mathematical term, since that acceleration may not
be toward any particular center point, there is
merely a departure from inertial motion.

Essentially all "forces" that do not involve
physical contact or interaction are inertial forces,
gravity, Coriolis, centrifugal.
But it took General Relativity to show that
gravity is an inertial force.

Ken Fischer

me...@cars3.uchicago.edu

unread,
May 15, 1997, 3:00:00 AM5/15/97
to

In article <8637016...@dejanews.com>, mahipa...@orbital.fsd.com writes:
>In article <EA564...@midway.uchicago.edu>,
> me...@cars3.uchicago.edu wrote:
>>
>> In article <5lamfi$2...@news2.ic.net>, "Peter Diehr" <pdi...@mail.ic.net>
>writes:
>> >
>> >You are correct to question this, Mati! Einstein praised Newton many
>> >times. In one of his brief essays, he says something like "Newton
>> >accomplished as much as was possible for a first class mind, given the
>> >developments of the age".
>> >
>> >I see no criticism of Newton in this.
>> >
>> Neither do I. But it seems that we're doomed to struggle, eternally,
>> against the naive view of the history of science as a process where
>> every generation somebody proves that all his predecessors were just
>> stupid. It is but a half year since the infamous "Newton was wrong"
>> thread and here we go again.
>
>Both of you need to get a perspective. Calm down. You write as if I said
>Einstein made personal comments about Newton. The one statement of mine
>above is just: "Einstein's work proved that Newton ..."
>
>It's common practice to contract sentences like that once in a while.
>
And I called this remark "a silly bit of childishness". This is my
common practice.

>Sorry if the statement bothers you. But you need remember that due to
>natural advances in experimental sciences, it is more likely than not
>that future physicists will say *we* *TODAY* also didn't know physics
>very well.

And this is another silly bit of childishness. Guess what, physicist
today do not say that Newton or Laplace or Huygenss didn't know
physics very well. They do not say it about Archimedes, either,
though he lived more then 20 centuries ago. Though we did progress
beyond their stuff, it keeps serving as a foundation to what we're
doing. It is not something that became obsolete, of interest only to
historians of science.

>
>Hecht, historically Newton wasn't all that nice a guy either. Apparently
>he screwed both Hooke and Leibniz.

Was somebody talking about "niceness".


>
>Not that Newton's or anyone's personal actions and attitudes are of any
>consequence to F=ma or G_{un}=T_{un} or H|v>=E|v> or ...
>

Exactly.

mahipa...@orbital.fsd.com

unread,
May 15, 1997, 3:00:00 AM5/15/97
to

In article <EA6r6...@midway.uchicago.edu>,
me...@cars3.uchicago.edu wrote:
>
> mahipa...@orbital.fsd.com writes:
[trim]

> >A specific example would be the precession of Mercury blunder.
>
> Oh, yeah. The "huge" precession, a whooping 10^(-8) relative change.
> How could they miss it?-) "Guess what, Ma, we've been lied to all
> these years. Turns out that Mt. Everest is half a milimeter lower
> than what the books say". Took another two hundred years after
> Newton before it could've even be measured.
> >
> >Q: Who was experimentally more correct, Einstein or Newton wrt Mercury's
> >data?
>
> How the hell can somebody be correct or incorrect wrt data that
> doesn't exist at the time he does his work?

I see your perspective and raise you to higher ground. Like the scenery?
How do you like being a free entity traveling as if separated from Time's
Trap? ;-)

Of course Newton had no data wrt Mercury's precession. But ultimatley,
that's not the point. Because, even *if* the data were available, Newton
Dynamics would've made the same mathematical predictions that ND yield
today and are algorthmically committed to predicting for all tomorrows.

Indeed, Newton was not obligated to model phenomena that wasn't
*available*.

> >As many are well aware, "forces" really aren't anymore since we now have
> >"geodesics".
>
> Really? Describe EM forces in terms of geodesics, please.

Would that I could, Friend. Would that I could. Maybe by next year.

But isn't that part of the problem? That charged particles don't seem to
warp spacetime as mass-energy particles cause spacetime to bend? Hence
this is one of the reasons "gravity" is so phenomenologically (I just
twisted my tongue saying that word) different? But wait! Wouldn't the
"geodesics" camp argue that the curvature is influenced by the presence
of {+q,-q} charges thereby effectively accounting for the significantly
distinct trajectories between particles with charge and those without?
Mathematically, there's seemingly unlimited maneuvering room. Weasels!

Bascially, if F=ma and thinking in "forces" works to effectively model
physical phenomena to within acceptable experimental bounds with
predetermined error bars, then the intimidating rhetorical "geodesics" is
highly over rated.

Of course, I have no quarms with studying and thinking in both ways.

mahipa...@orbital.fsd.com

unread,
May 15, 1997, 3:00:00 AM5/15/97
to

In article <shepard-1405...@macrls.tcg.anl.gov>,

she...@tcg.anl.gov (Ron Shepard) wrote:
>
> In article <5lb6ii$o...@vixen.cso.uiuc.edu>, prus...@staff.uiuc.edu (John
> E. Prussing) wrote:
>
> [...]
> >>> >> she...@tcg.anl.gov (Ron Shepard) writes:
> >>> >[trim]
> >>> >> >BTW, it seems to me that the centrifugal force
> >>> >> >arises from the use of a polar coordinate system, not from a rotating
> >>> >> >reference frame.
> >
> > [...] As a
> >practical matter, for spacecraft traveling in our solar system, a
> >perfectly good inertial frame over a decade or so is one centered in
> >our sun and non-rotating with repsect to distant stars. [...]
>
> This is what I meant when I said that it is the polar coordinate system,
> not the rotating reference frame that gives rise to "centrifugal force".

I'm sorry if I seem like "I must sit you down and set you straight", but
i am just being polite by letting you know you are in error. What you've
said, now twice, is as wrong as it was the first time.

The *rotating* reference frame gives rise to the centrifugal force. A
frame can be rectangular and still be rotating. A frame can be polar and
not be rotating.

Any frame can be either rotating or nonrotating.
Rotating frames, whether polar or not, are inherently accelerating.
An inertial reference frame is a nonaccelerating frame.
Therefore, a rotating reference frame can never be inertial.
A translating frame can be inertial, provided it is NOT accelerating.
And in Nature, there are no observed absolute and pure inertial frames.

A "for all practical purposes inertial frame" is one which is STATIC for
time durations greater than the timescale of any particular problem being
analyzed. That's why the center of the Earth serves as a useful inertial
reference frame for studying manmade satelites which have orbital
lifespans of only a few years. The Vernal Equinox, the point of Aires,
hardly shifts at all over 50 years. So, practically, most inertial frames
are a matter of convenience and are not rigid axes in space or spacetime.

> This is the _standard_ treatement in fact. For example, see Marion's
> "Classical Dynamics" pp. 247-278 or so. The L^2/(2*mu*r^2) term is added
> to the potential energy, and the derivative of this w.r.t. "r" is called
> the "centrifugal force". Forces are often defined as derivatives of
> potential energy expressions, but in this case it is convenient to include
> this piece of the kinetic energy since L is a conserved quantity. Whether
> the "theta" part of the coordinate system is inertially fixed or rotating
> doesn't really enter the argument.

By this example of your's, you are making the classic mistake of
misleading yourself from extrapolating from a specific instance to a
general error.

By your argument, all centrifugal forces would simply vanish if we all
opted to just use cartesian coordinates.

Let's say you have a polar coordinate frame. In this, r = r |e_r> + 0
|e_theta> where "|>" is just a way to represent a vector. This is Dirac's
Notation. Let me make up fun nonsense as I go along. From Dirac, "D".
From "D" to "|>". See the broken geometric connection? ;-) Fonts are a
great way to learn geometry. Especially if one is into writing for
"appearance" sake. Now,

|e_r> is the unit vector in the r direction.
|e_theta> is the unit vector in the theta direction.

But any vector can be projected onto an infinity of coordinate bases.
This bit of mathematical knowledge is extremely useful in future studies
of QM, GR, and so on. To go from polar to rectangular coordinates:
(r,theta) maps to (x=r cos(theta), y=r sin(theta)). Which is the same as
saying:

r == r |e_r> + 0 |e_theta> = r*cos(theta) |e_x> + r*sin(theta) |e_y>

There are perhaps as many different but *equivalent representations* as
there are teachers with chalk marks on their hands from filling up
blackboards!

Basically, the same centrifugal force which exists in the polar reference
frame also exists (i.e., can be represented) equivalently in the
rectangular frame. This force also exists, however fictional, in an
infinity of other equiExistent but not used coordinate bases! The
"fictional" is indeed the tricky part.

Thousands, nay billions, of posters would jump to prove my explaination
wrong! They already have corrected my postings where they needed it. And
I thank them all for correcting my errors. This process is called
LEARNING. ;-) Provided I listen to the schmucks. [Double friendly
smiley].

me...@cars3.uchicago.edu

unread,
May 15, 1997, 3:00:00 AM5/15/97
to
... snip ...

>
>Of course Newton had no data wrt Mercury's precession. But ultimatley,
>that's not the point. Because, even *if* the data were available, Newton
>Dynamics would've made the same mathematical predictions that ND yield
>today and are algorthmically committed to predicting for all tomorrows.

True. Which does absolutely nothing to support your statement
"Einstein proved that Newton didn't know that much physics". Such
statement can be only justified when somebody, having at his hands the
information needed to generate a better model, comes with an inferior
one instead.

Newton came with the best model possible, given the information
available at his time. Two hundred years later Einstein, having more
and better information, came with a better one. And some time in the
future somebody, having yet better information, may come with the next
version. That's how things work. It is not a game of proving your
predecessors stupid but of building on what they did in order to get
yet further.


>>
>> >As many are well aware, "forces" really aren't anymore since we now have
>> >"geodesics".
>>
>> Really? Describe EM forces in terms of geodesics, please.
>
>Would that I could, Friend. Would that I could. Maybe by next year.

Thus, as by now, your statement above turns out to be quite
inaccurate.


>
>But isn't that part of the problem? That charged particles don't seem to
>warp spacetime as mass-energy particles cause spacetime to bend? Hence
>this is one of the reasons "gravity" is so phenomenologically (I just
>twisted my tongue saying that word) different? But wait! Wouldn't the
>"geodesics" camp argue that the curvature is influenced by the presence
>of {+q,-q} charges thereby effectively accounting for the significantly
>distinct trajectories between particles with charge and those without?

No, it doesn't work like this. Whatever the curvature is, it is a
single value at each location. If you start to introduce a whole
bunch of them, it becomes "epicycle like".


>
>Bascially, if F=ma and thinking in "forces" works to effectively model
>physical phenomena to within acceptable experimental bounds with
>predetermined error bars, then the intimidating rhetorical "geodesics" is
>highly over rated.

Well, I would consider the possibility of reducing all interaction to
space-time properties intriguing. GR is a step in this direction but
if you were under the impression that all forces have been reduced to
geodesic, it just ain't so.

Mike Norris

unread,
May 16, 1997, 3:00:00 AM5/16/97
to

In article <EA8Bu...@iglou.com>, Ken Fischer

<URL:mailto:kefi...@iglou.com> wrote:
>
> Xtremethug (xtrem...@aol.com) wrote:
> : My teacher put it this way:
> : Centrifugal force is ficticious. What is pushing the "object"
> : outwards is the law of inertia (straight line motion unless acted upon by
> : an outside force) and of course that outside force is centrepital force
>
> That is correct, no physicist will argue with that.

Yes they would ... nothing is pushing the object outwards.
But the first sentence is correct.

> But inertia is real, and perhaps one of the most powerful forces
> in the universe.

Inertia is there and very real but it is not a force, it doesn't push,
it just is!

> The arguement has been about the use of the word
> "centrifugal" force, some people seem to feel the word
> should not be used.

Absolutely ... it is extremely confusing ... terminonolgy is VERY important
it means 'centre fleeing' ...
motion/force away from the centre ...
no such force exists in any frame!

> Ken Fischer


I've been away and just caught up with this thread ...

It seems to me that there are an awful lot of confused people out there,
confused by use of this particular term.

Try this one for size .....

2 equal mass binary stars orbiting each other ...

nothing else around ... it must be true somewhere out there!

it's pretty obvious where the centripetal force is and what generates it,
even if our descriptors are incomplete.

but centrifugal !!!!!!!!!!!!!!!!!!!!! ... nowhere to be seen or felt !!!!!!!!!!!!!!!

If it's not there it's not anywhere!

mi...@mikenorr.demon.co.uk
The English Lake District.


Ron Shepard

unread,
May 16, 1997, 3:00:00 AM5/16/97
to

In article <8637311...@dejanews.com>, mahipa...@orbital.fsd.com wrote:

[...]


>I'm sorry if I seem like "I must sit you down and set you straight", but
>i am just being polite by letting you know you are in error. What you've
>said, now twice, is as wrong as it was the first time.
>
>The *rotating* reference frame gives rise to the centrifugal force. A
>frame can be rectangular and still be rotating. A frame can be polar and

>not be rotating. [...]

This is true, but what rotating frame has been defined that leads to
centrifugal force?

[regarding the definition of "centrifugal force" in a nonrotating frame...]


>> This is the _standard_ treatement in fact. For example, see Marion's
>> "Classical Dynamics" pp. 247-278 or so. The L^2/(2*mu*r^2) term is added
>> to the potential energy, and the derivative of this w.r.t. "r" is called
>> the "centrifugal force". Forces are often defined as derivatives of
>> potential energy expressions, but in this case it is convenient to include
>> this piece of the kinetic energy since L is a conserved quantity. Whether
>> the "theta" part of the coordinate system is inertially fixed or rotating
>> doesn't really enter the argument.

My point here was that "centrifigal force" is usually defined as a
derivative of a potential energy function which, in turn, is defined in
terms of an inertial polar coordinate system. It is the polar coordinate
system that leads to this definition. Since it is inertial, nonrotating
in this case, it cannot be due to the fact that it is a rotating
coordinate system. But the L^2/(2*mu*r^2) contribution to the "potential"
depends on L^2 which is a conserved quantity, a fixed value in other
words. The value of this fixed number would change in a rotating
coordinate system, but it would still be a constant and there would only
be the "r" dependence when taking the derivatives, there is no "theta"
dependence.

>By this example of your's, you are making the classic mistake of
>misleading yourself from extrapolating from a specific instance to a
>general error.

Perhaps so, but I have given an example of how "centrifugal force" is
defined in a straightforward way using an inertial polar coordinate
system. Maybe I could see your point of view if you would demonstrate how
a rotating coordinate system (rectangular or polar) leads to a simpler
definition of "centrifugal force"?

>By your argument, all centrifugal forces would simply vanish if we all
>opted to just use cartesian coordinates.

In a way, yes. It depends on how the derivatives of the potential
function are defined. It is these derivatives that define the forces,
right? It also depends on moving that piece of kinetic energy over into
the potential energy when you take the derivatives.

>Let's say you have a polar coordinate frame. In this, r = r |e_r> + 0
>|e_theta> where "|>" is just a way to represent a vector. This is Dirac's
>Notation. Let me make up fun nonsense as I go along. From Dirac, "D".
>From "D" to "|>". See the broken geometric connection? ;-) Fonts are a
>great way to learn geometry. Especially if one is into writing for
>"appearance" sake.

I've heard another story about Dirac notation. Brassieres were first
becoming popular in France about the time that a convenient notation for
scalar products were found to be needed in quantum mechanics. Dirac was a
young man at the time and was particularly good at thinking up notational
shortcuts. So out of several possibilities, we have ended up using the
<|> notation where <| is the "bra" and |> is the "ket". So not only is it
a verbal pun "bra"+"ket"=bracket of sorts, it is also a visual pun on the
newfangled undergarmets that were just beginning to be called "bras".

>Now,
>
>|e_r> is the unit vector in the r direction.
>|e_theta> is the unit vector in the theta direction.
>
>But any vector can be projected onto an infinity of coordinate bases.
>This bit of mathematical knowledge is extremely useful in future studies
>of QM, GR, and so on. To go from polar to rectangular coordinates:
>(r,theta) maps to (x=r cos(theta), y=r sin(theta)). Which is the same as
>saying:
>
>r == r |e_r> + 0 |e_theta> = r*cos(theta) |e_x> + r*sin(theta) |e_y>
>
>There are perhaps as many different but *equivalent representations* as
>there are teachers with chalk marks on their hands from filling up
>blackboards!
>
>Basically, the same centrifugal force which exists in the polar reference
>frame also exists (i.e., can be represented) equivalently in the
>rectangular frame. This force also exists, however fictional, in an
>infinity of other equiExistent but not used coordinate bases! The
>"fictional" is indeed the tricky part.

But if you differentiate the potential w.r.t. the |e_x> and |e_y>
components, there is indeed no "centrifugal force" that shows up. It is
when you represent it in terms of r and theta, and add in the conserved
L^2 contribution from the kinetic energy into the potential, and then
differentiate w.r.t. "r" that the "centrifugal force" looks like a force.

But the vector transformations are just a tangent (an unintentional pun
:-) to the original question about the nature of "centrifugal force".
Does it arise from a rotating coordinate system, or from the choice of a
polar coordinate system? I've pointed out that the usual textbook
derivation uses the latter approach. There are of course equivalent
representations of this force in other coordinate systems, including
rotating ones, but a rotating coordinate system is not necessary to
understand, or to derive in a simple way, the definition of this fictional
"centrifugal force".

$.02 -Ron Shepard

me...@cars3.uchicago.edu

unread,
May 16, 1997, 3:00:00 AM5/16/97
to

>Try this one for size .....
>
> 2 equal mass binary stars orbiting each other ...
>
> nothing else around ... it must be true somewhere out there!
>
>it's pretty obvious where the centripetal force is and what generates it,
>even if our descriptors are incomplete.
>
> but centrifugal !!!!!!!!!!!!!!!!!!!!! ... nowhere to be seen or felt !!!!!!!!!!!!!!!
>
>If it's not there it's not anywhere!
>

Now, describe the same thing in the rotating frame of the two masses.

Mike Norris

unread,
May 18, 1997, 3:00:00 AM5/18/97
to

In article <EAAnD...@midway.uchicago.edu>,

<URL:mailto:me...@cars3.uchicago.edu> wrote:
>
> In article <ant16211...@mikenorr.demon.co.uk>, Mike Norris <mi...@mikenorr.demon.c
> o.uk> writes:
>
> >Try this one for size .....
> >
> > 2 equal mass binary stars orbiting each other ...
> >
> > nothing else around ... it must be true somewhere out there!
> >
> >it's pretty obvious where the centripetal force is and what generates it,
> >even if our descriptors are incomplete.
> >
> > but centrifugal !!!!!!!!!!!!!!!!!!!!! ... nowhere to be seen or felt !!!!!!!!
> !!!!!!!
> >
> >If it's not there it's not anywhere!
> >
> Now, describe the same thing in the rotating frame of the two masses.

There is no point to this because there will be no centre to flee from!
So you do it ...without using the term centrifugal ... and then you'll be there!

me...@cars3.uchicago.edu

unread,
May 18, 1997, 3:00:00 AM5/18/97
to

In article <ant18002...@mikenorr.demon.co.uk>, Mike Norris <mi...@mikenorr.demon.co.uk> writes:
>In article <EAAnD...@midway.uchicago.edu>,

>> >
>> > but centrifugal !!!!!!!!!!!!!!!!!!!!! ... nowhere to be seen or felt !!!!!!!!
>> !!!!!!!
>> >
>> >If it's not there it's not anywhere!
>> >
>> Now, describe the same thing in the rotating frame of the two masses.
>
>There is no point to this because there will be no centre to flee from!
>So you do it ...without using the term centrifugal ... and then you'll be there!
But of course there is a center. It is the center of mass of the
system. And, there is the force of gravity pulling them together,
which from the point of view of each of them is pulling towards the
center of mass. Yet, they don't fall towards the center of mass.
Well?

Ken Fischer

unread,
May 18, 1997, 3:00:00 AM5/18/97
to

Mike Norris (mi...@mikenorr.demon.co.uk) wrote:
: mailto:me...@cars3.uchicago.edu> wrote:

: >Mike Norris <mi...@mikenorr.demon.c o.uk> writes:
: > >Try this one for size .....
: > >
: > > 2 equal mass binary stars orbiting each other ...
: > >
: > > nothing else around ... it must be true somewhere out there!
: > >
: > >it's pretty obvious where the centripetal force is and what generates it,
: > >even if our descriptors are incomplete.
: > >
: > > but centrifugal !!!!!!!!!!!!!!!!!!!!! ...
: > > nowhere to be seen or felt !!!!!!!!!!!!!!
: > >
: > >If it's not there it's not anywhere!

: >
: > Now, describe the same thing in the rotating frame of the two masses.

: There is no point to this because there will be no centre to flee from!
: So you do it ...without using the term centrifugal ... and then you'll
: be there!

"No center to flee from", is that a technical
term? What can you have against the word "centrifugal"?
I think I see the problem, in the case of binary
stars, you assume a centripetal force, and call it gravity.
I think Einstein would have disagreed.
Orbiting objects are in inertial motion, that is
what non-acceleration means, and the General Theory of
Relativity is based on that.

You keep talking about "centripetal acceleration"
in the case of the string and stone. If an automobile
goes around a curve, is it "centripetal", or simply
"lateral"?

Ken Fischer

mahipa...@orbital.fsd.com

unread,
May 18, 1997, 3:00:00 AM5/18/97
to

In article <EA8nq...@midway.uchicago.edu>,[trim]

> >Of course Newton had no data wrt Mercury's precession. But ultimatley,
> >that's not the point. Because, even *if* the data were available, Newton
> >Dynamics would've made the same mathematical predictions that ND yield
> >today and are algorthmically committed to predicting for all tomorrows.
>
> True. Which does absolutely nothing to support your statement
> "Einstein proved that Newton didn't know that much physics". Such
> statement can be only justified when somebody, having at his hands the
> information needed to generate a better model, comes with an inferior
> one instead.

Me1: But Einstein's proved Newton didn't know physics all that well.
Me2: "Einstein proved that Newton didn't know that much physics"
Me3: [Silence]

Let's not have sematic quibbles over a minor statement that I made out of
the blue, as if in an informal conversation. Clearly I meant "Einstein's
work with GR and experimental evidence of Mercury's perihelion percession
and stellar gravitational lensing imply that Newtonian Dynamics have a
limited scope of applicability."

Physics knowledge is conditional on the availability of experimental
observations. Hence, by the time of Einstein, new phenomena that Newton
wasn't even privy to became available.

Now imagine the darker side of this situation: if experimental evidence
is not shared or publically available. For instance, where would Kepler
be if Brahe refused to share his planetary observations? How about the
same situation in modern work environments between experimentalist and
theoreticians?

> Newton came with the best model possible, given the information
> available at his time. Two hundred years later Einstein, having more
> and better information, came with a better one. And some time in the
> future somebody, having yet better information, may come with the next
> version. That's how things work. It is not a game of proving your
> predecessors stupid but of building on what they did in order to get
> yet further.

I am certain you and I agree here almost exactly. Newton was a very
capable physicist for _his_ time. If Newton were alive today, he might
not make the grade! Late 20th Century physics is tough I tell ya. I don't
want to sound as if denegrating Newton, for I am not. It just so happens
that new phenomena were discovered as measurements improved in the
natural progression of time. The new and better data warranted the
development of both GR and QM.

> >> >As many are well aware, "forces" really aren't anymore since we now have
> >> >"geodesics".
> >>
> >> Really? Describe EM forces in terms of geodesics, please.
> >
> >Would that I could, Friend. Would that I could. Maybe by next year.
>
> Thus, as by now, your statement above turns out to be quite
> inaccurate.

It's not "my" statement. It's a reflection of what readers are lead to
believe when they read some advanced, and some not so advanced, physics
literature. For reference: "Gravity is not a force." I'm sure many have
read this before.

> >But isn't that part of the problem? That charged particles don't seem to
> >warp spacetime as mass-energy particles cause spacetime to bend? Hence
> >this is one of the reasons "gravity" is so phenomenologically (I just
> >twisted my tongue saying that word) different? But wait! Wouldn't the
> >"geodesics" camp argue that the curvature is influenced by the presence
> >of {+q,-q} charges thereby effectively accounting for the significantly
> >distinct trajectories between particles with charge and those without?
>
> No, it doesn't work like this. Whatever the curvature is, it is a
> single value at each location. If you start to introduce a whole
> bunch of them, it becomes "epicycle like".

Exactly. It does seem to become epicyclic-like. But there's nothing
really wrong with this approach. We're doing the best guess that we
currently can?

The mass of a He atom and an ionized He atom are nearly identical. But
the trajectories these two particles follow in the presence of an electro
magnetic field are dramatically distinct from one another. The geodesic
model, if single valued, clearly seems to lack the charged particle
interations. Help. ;-)

> >Bascially, if F=ma and thinking in "forces" works to effectively model
> >physical phenomena to within acceptable experimental bounds with
> >predetermined error bars, then the intimidating rhetorical "geodesics" is
> >highly over rated.
>
> Well, I would consider the possibility of reducing all interaction to
> space-time properties intriguing. GR is a step in this direction but
> if you were under the impression that all forces have been reduced to
> geodesic, it just ain't so.

It's nice to read that the transformation from "forces to geodesics" is
incomplete. I get the impression this f2g-transformation is in the works,
however. Isn't that why John Baez can say that "_all_ forces are made-up"?

I'm not anti- or pro- physics. I'm just trying to understand "physics".
Peace. Believe me, when I find something in physics that upsets my
personal worldview, I'll simply burn all my physics books real good. OK?
;-)

me...@cars3.uchicago.edu

unread,
May 18, 1997, 3:00:00 AM5/18/97
to

In article <ant18194...@mikenorr.demon.co.uk>, Mike Norris <mi...@mikenorr.demon.co.uk> writes:
>In article <EACpA...@midway.uchicago.edu>,
><URL:mailto:me...@cars3.uchicago.edu> wrote:
>>
>> In article <ant18002...@mikenorr.demon.co.uk>, Mike Norris <mi...@mikenorr.demon.c
>> o.uk> writes:
>> >In article <EAAnD...@midway.uchicago.edu>,

>> >> >
>> >> > but centrifugal !!!!!!!!!!!!!!!!!!!!! ... nowhere to be seen or felt !!!!!
>> !!!
>> >> !!!!!!!

>> >> >
>> >> >If it's not there it's not anywhere!
>> >> >
>> >> Now, describe the same thing in the rotating frame of the two masses.
>> >
>> >There is no point to this because there will be no centre to flee from!
>> >So you do it ...without using the term centrifugal ... and then you'll be there!
>> But of course there is a center. It is the center of mass of the
>> system. And, there is the force of gravity pulling them together,
>> which from the point of view of each of them is pulling towards the
>> center of mass. Yet, they don't fall towards the center of mass.
>> Well?
>
>O.K. ... I think we're getting there but still at cross purposes!
>
>I think the point I'm trying to make that we should not transfer definitions
>made in one frame to another frame. Both the words centripetal and centrifugal
>have been defined in non rotating frameworks and cannot be sensibly
>used in a rotating one.

No, not at all. It is exactly within the rotating frame where the
term "centrifugal" applies. The centrifugal force is what you need to
add to the description of the situation so as to maintain the validity
of Newton's Second Law.

... snip ...
>
>So O.K. invent the forces necessary for the given situation but be very careful
>not to use the same term in different frames with different meanings.
>What exists in one frame may not exist in another.

Right. That's why we make an effort to distinguish "inertial forces"
which are frame dependent, from "true physical forces" which are not.
And, by a proper selection of a frame, the inertial forces can be
eliminated all together. The only confusion is in the thinking that
since such elimination is possible it "must" always be done and
failing to do it is an error. This is simply not true.
>
>My point is that for any introductory work in the frame we live in, especially
>studying local effects, the use of the term centifugal is simply wrong,
>it is non-existent.

Sorry, I disagree. This is false.
>
>In common use it is used very casually and incorrectly.

That I agree with. But, this just meansthat it should be taught
correctly, not declared "nonexistant" so as to avoid errors. Fighting
a fallacy with another fallacy is a bad idea.

mahipa...@orbital.fsd.com

unread,
May 18, 1997, 3:00:00 AM5/18/97
to

In article <EA8CF...@midway.uchicago.edu>,
> And I called this remark "a silly bit of childishness". This is my
> common practice.

I don't mind, at all, being called childish. That's fine by me for I am
most certainly playful. My style of interacting on Usenet is surely not
everyone's cup of tea. But hey, I gotta be Me. Even where the medium is
ASCII streams. Most of my gestures get lost in the translation. I'm sure
I lose credibility in the eyes of some, and at the same time other people
may take me seriously. Perception is in the eye of the beholder. ;-)

> >Sorry if the statement bothers you. But you need remember that due to
> >natural advances in experimental sciences, it is more likely than not
> >that future physicists will say *we* *TODAY* also didn't know physics
> >very well.
>
> And this is another silly bit of childishness. Guess what, physicist
> today do not say that Newton or Laplace or Huygenss didn't know
> physics very well. They do not say it about Archimedes, either,
> though he lived more then 20 centuries ago. Though we did progress
> beyond their stuff, it keeps serving as a foundation to what we're
> doing. It is not something that became obsolete, of interest only to
> historians of science.

No we don't say those things directly. But we sincerely and honestly have
to sit down and contemplate the possibility of what all our great
teachers may or may not have missed. Thinking is a tough business and not
for the faint hearted.

What are *we* missing that only future physicists would be fortunate
enough to think? Can there *be* future physicsts without the current
thoughts influencing, whether positively or not, them?

Again, my being childish and playful is not just grounds for associating
me with the pomo-camp who inspired the "Newton was wrong" nonsense.

[snip]


> >Not that Newton's or anyone's personal actions and attitudes are of any
> >consequence to F=ma or G_{un}=T_{un} or H|v>=E|v> or ...
> >
> Exactly.

Yep. ;-) Even if I do look as if to agree with myself. Silly me.

me...@cars3.uchicago.edu

unread,
May 18, 1997, 3:00:00 AM5/18/97
to

In article <ant18193...@mikenorr.demon.co.uk>, Mike Norris <mi...@mikenorr.demon.co.uk> writes:
>
>Both centripetal and centrifugal have been defined in a non-rottaing
>frame and must be redifined or preferably not used elsewhere.

This is wrong.

Mike Norris

unread,
May 18, 1997, 3:00:00 AM5/18/97
to

In article <EACpA...@midway.uchicago.edu>,
<URL:mailto:me...@cars3.uchicago.edu> wrote:
>
> In article <ant18002...@mikenorr.demon.co.uk>, Mike Norris <mi...@mikenorr.demon.c
> o.uk> writes:
> >In article <EAAnD...@midway.uchicago.edu>,
> >> >
> >> > but centrifugal !!!!!!!!!!!!!!!!!!!!! ... nowhere to be seen or felt !!!!!
> !!!
> >> !!!!!!!
> >> >
> >> >If it's not there it's not anywhere!
> >> >
> >> Now, describe the same thing in the rotating frame of the two masses.
> >
> >There is no point to this because there will be no centre to flee from!
> >So you do it ...without using the term centrifugal ... and then you'll be there!
> But of course there is a center. It is the center of mass of the
> system. And, there is the force of gravity pulling them together,
> which from the point of view of each of them is pulling towards the
> center of mass. Yet, they don't fall towards the center of mass.
> Well?

O.K. ... I think we're getting there but still at cross purposes!

I think the point I'm trying to make that we should not transfer definitions
made in one frame to another frame. Both the words centripetal and centrifugal
have been defined in non rotating frameworks and cannot be sensibly
used in a rotating one.

An example of this would be Newtons First Law of Motion which is NOT
the same in a rotating frame as it is in a non-rotating one ...
... let me see if I remember...

Non-rotating frame ... Any object will remain in its state of rest or of uniform
motion in a straight line unless acted upon by some external force.

Rotating frame ... A body remains in a state of rest if and only if a force
acts on it, this force being directed towards a fixed point and proportional
to the distance from that point.

... I think that's right!

So O.K. invent the forces necessary for the given situation but be very careful
not to use the same term in different frames with different meanings.
What exists in one frame may not exist in another.

My point is that for any introductory work in the frame we live in, especially

studying local effects, the use of the term centifugal is simply wrong,
it is non-existent.

In common use it is used very casually and incorrectly.
The concept of centripetal force/acceleration has not even been heard of.
This leads to poor understanding and problem solving becomes impossible.

I can dig up some examples if you want!

me...@cars3.uchicago.edu

unread,
May 18, 1997, 3:00:00 AM5/18/97
to
... snip ...

>
>Let's not have sematic quibbles over a minor statement that I made out of
>the blue, as if in an informal conversation. Clearly I meant "Einstein's
>work with GR and experimental evidence of Mercury's perihelion percession
>and stellar gravitational lensing imply that Newtonian Dynamics have a
>limited scope of applicability."

But of course. No sensible person will quible with this.


>
>Physics knowledge is conditional on the availability of experimental
>observations. Hence, by the time of Einstein, new phenomena that Newton
>wasn't even privy to became available.
>
>Now imagine the darker side of this situation: if experimental evidence
>is not shared or publically available. For instance, where would Kepler
>be if Brahe refused to share his planetary observations? How about the
>same situation in modern work environments between experimentalist and
>theoreticians?

Such situations may indeed occur. But, what one person found, sooner
or later (usually sooner) another one will. In the grand scheme of
things it really isn't that important at what specific date and by
what specific individual something was done.


>
>> Newton came with the best model possible, given the information
>> available at his time. Two hundred years later Einstein, having more
>> and better information, came with a better one. And some time in the
>> future somebody, having yet better information, may come with the next
>> version. That's how things work. It is not a game of proving your
>> predecessors stupid but of building on what they did in order to get
>> yet further.
>
>I am certain you and I agree here almost exactly. Newton was a very
>capable physicist for _his_ time. If Newton were alive today, he might
>not make the grade!

Frankly, I doubt it. Seeing how much Newton managed to do with what
he had available, I'm reasonably sure that would he live nowadays, his
contribution would've been enormous. Different, mind you, but
enormous. Still, arguing over what might've been is pretty pointless.


>
>> >> >As many are well aware, "forces" really aren't anymore since we now have
>> >> >"geodesics".
>> >>
>> >> Really? Describe EM forces in terms of geodesics, please.
>> >
>> >Would that I could, Friend. Would that I could. Maybe by next year.
>>
>> Thus, as by now, your statement above turns out to be quite
>> inaccurate.
>
>It's not "my" statement. It's a reflection of what readers are lead to
>believe when they read some advanced, and some not so advanced, physics
>literature. For reference: "Gravity is not a force." I'm sure many have
>read this before.

Sure. Now. compare the two statements "Mahipal isn't an aardvark" and
"there are no aardvarks in the world". Do you see a slight difference
between them?

So, one should be a bit careful before jumping to broad sweeping
generalizations based on limited statements. Gravity is not the only
thing around, you know. Beyond this, one should be careful
interpreting statements like "X is (or isn't) a force". Their meaning
depends on how broad or narrow our definition of a force is.
Interestingly enough this ties back into the issue of centrifugal
forces which started this debate. As i wrote elsewhere, we
distinguish between "real physical forces" which can be ultimately
traced to some particle-particle interaction and "inertial forces"
which result from the selection of a specific reference frame to
describe the dynamics and can be fully eliminated by choosing another
reference frame. Mind you, those "inertial forces" still have very
real effects relative to frames where they're not eliminated. So,
anyway, when we say "gravity is not a force" what we mean is "gravity
belongs in the inertial forces category, not in the real physical
forces one".


>
>> >But isn't that part of the problem? That charged particles don't seem to
>> >warp spacetime as mass-energy particles cause spacetime to bend? Hence
>> >this is one of the reasons "gravity" is so phenomenologically (I just
>> >twisted my tongue saying that word) different? But wait! Wouldn't the
>> >"geodesics" camp argue that the curvature is influenced by the presence
>> >of {+q,-q} charges thereby effectively accounting for the significantly
>> >distinct trajectories between particles with charge and those without?
>>
>> No, it doesn't work like this. Whatever the curvature is, it is a
>> single value at each location. If you start to introduce a whole
>> bunch of them, it becomes "epicycle like".
>
>Exactly. It does seem to become epicyclic-like. But there's nothing
>really wrong with this approach. We're doing the best guess that we
>currently can?

No problem here. But the best should still be better then the
previous one. If you "geometrize" EM through the introduction of
multivalued metric and don't get out of it anything more than what you
already know, then you've just introduced an enormous complexity for
no good purpose. If all GR was doing was to recapitulate Newtonian
Gravity using different terms, its value would've been rather
questionable.
>
... snip ...

>It's nice to read that the transformation from "forces to geodesics" is
>incomplete. I get the impression this f2g-transformation is in the works,
>however. Isn't that why John Baez can say that "_all_ forces are made-up"?

No, that's not the reason. But, I'll leave it to him to answer, if he
wants.


>
>I'm not anti- or pro- physics. I'm just trying to understand "physics".
>Peace. Believe me, when I find something in physics that upsets my
>personal worldview, I'll simply burn all my physics books real good. OK?
>;-)

And then, next time you're gluing something and need a heavy object to
keep it in place till the glue dries, what you're going to do? Oh,
well, there are always dictionaries.

me...@cars3.uchicago.edu

unread,
May 18, 1997, 3:00:00 AM5/18/97
to

In article <8639877...@dejanews.com>, mahipa...@orbital.fsd.com writes:
>In article <EA8CF...@midway.uchicago.edu>,

> me...@cars3.uchicago.edu wrote:
>>
>>
>> And this is another silly bit of childishness. Guess what, physicist
>> today do not say that Newton or Laplace or Huygenss didn't know
>> physics very well. They do not say it about Archimedes, either,
>> though he lived more then 20 centuries ago. Though we did progress
>> beyond their stuff, it keeps serving as a foundation to what we're
>> doing. It is not something that became obsolete, of interest only to
>> historians of science.
>
>No we don't say those things directly. But we sincerely and honestly have
>to sit down and contemplate the possibility of what all our great
>teachers may or may not have missed. Thinking is a tough business and not
>for the faint hearted.

You still don't understand what I'm trying to say. You can't miss
what's not there. A "miss" refers to a situation where the
information needed to reach the "right" conclusion is available and
ignored or misinterpreted. In the absence of such situation talking
about a "miss" is a nonsense.

Sometimes a situation occurs where the information available allows
for more than one interpretation. In such case different people may
pick different interpretations and guess what, none of them is
"wrong". And if later new information becomes available, sufficient
to disqualify all but one of the previously existing interpretations,
this makes the one interpretation that survived "right" but it doesn't
make those who previously opted for other interpretations "wrong".

Mike Norris

unread,
May 18, 1997, 3:00:00 AM5/18/97
to

In article <EACyE...@iglou.com>, Ken Fischer
<URL:mailto:kefi...@iglou.com> wrote:

> "No center to flee from", is that a technical term?

If you like ... just a literal translation!

>What can you have against the word "centrifugal"?

Centrifugal effects don't exist from where I'm looking!

> I think I see the problem, in the case of binary
> stars, you assume a centripetal force, and call it gravity.

Not quite ... I see gravity and this provides the centripetal force.

> I think Einstein would have disagreed.

Yes I understand that.

> Orbiting objects are in inertial motion, that is
> what non-acceleration means, and the General Theory of
> Relativity is based on that.

From where I'm looking orbital objects are continuously being accelerated.

> You keep talking about "centripetal acceleration"
> in the case of the string and stone.
>If an automobile goes around a curve, is it "centripetal", or simply
> "lateral"?

The friction between the road and the tyre provides a centripetal force
which causes the car to accelerate, describing a circular path OR continue
in a striaght line into that tree if the frictional force is removed ... say
... by a patch of water!

Both centripetal and centrifugal have been defined in a non-rottaing frame and must
be redifined or preferably not used elsewhere.

> Ken Fischer

Jim Carr

unread,
May 19, 1997, 3:00:00 AM5/19/97
to

prus...@staff.uiuc.edu (John E. Prussing) wrote:
}
} [...] As a
} practical matter, for spacecraft traveling in our solar system, a
} perfectly good inertial frame over a decade or so is one centered in
} our sun and non-rotating with repsect to distant stars. [...]

she...@tcg.anl.gov (Ron Shepard) writes:
>
>This is what I meant when I said that it is the polar coordinate system,
>not the rotating reference frame that gives rise to "centrifugal force".

The *origin* of the frame says nothing about the way you set up
the coordinates. All of those are equivalent. The physics is
simpler in some than others, but that is all.

>This is the _standard_ treatement in fact. For example, see Marion's
>"Classical Dynamics" pp. 247-278 or so. The L^2/(2*mu*r^2) term is added
>to the potential energy, and the derivative of this w.r.t. "r" is called
>the "centrifugal force".

But this is not *caused* by the use of those coordinates. You can
do the whole think in cartesian coordinates as well, but not as easily.

> ... but in this case it is convenient ...
^^^^^^^^^^

Ron Shepard

unread,
May 19, 1997, 3:00:00 AM5/19/97
to

In article <EAEw6...@iglou.com>, kefi...@iglou.com (Ken Fischer) wrote:

[...]
> Take a look at video from Mir or the Shuttle,
>it is pretty obvious the occupants do not feel any
>acceleration, and I don't care to hear about gravity
>pulling and centrifugal force being equal, I don't
>like physics with the same terms on both sides of the
>formulas. [...]

What kind of acceleration are they supposed to "feel"? They are in free
fall! Every atom in every molecule in their body is being acclerated, but
so is everything else in their immediate surroundings, so they don't
"feel" any thing because there is nothing being forced against them.
Indeed, if by some act of magic, the centrepital force due to gravity were
turned off for the occupants, but left on for their surroundings, then
they would feel a force as the outside of the shuttle pushed against them,
or as they tried to pick up an object within the station. But gravity
doesn't work this way, they are all accelerated together, just as if they
were in free-fall going straight towards the earth surface. In this
latter case, only linear acceleration is involved, so these vague
references to "rotating coordinate systems" would clearly not apply to
this situation, just as they are not required (and this has been shown
explicitly in this thread) to explain "centrifugal force" in the usual
case of orbiting bodies. In the linear acceleration case, the occupants
would feel a force only when the outside of the station begin to feel the
frictional force from the air, or neglecting that, when the station
collided with the land or water at the earth's surface. The occupants
must be subjected to a different force than their surroundings in order
for them to "feel" anything. If there were a big tunnel bored through the
earth at the right location, then they would fall clear through the earth,
exit on the other side, and continue up into space again, and the
occupants would not "feel" acceleration at any time along the way. The
only difference in the orbital case and the free fall case, in this
respect, is that no tunnel is required in the orbital case. But it is
free fall nonetheless, and the occupants are accelerated in the same way,
and yet "feel" nothing. By the way, this applies also to elliptical
orbits where "r" and angular velocity change with time, and not just to
circular orbits where they are constant, or to the linear case in which
"r" changes but theta is constant.

$.02 -Ron Shepard

Ken Fischer

unread,
May 19, 1997, 3:00:00 AM5/19/97
to

Mike Norris (mi...@mikenorr.demon.co.uk) wrote:

: Ken Fischer (kefi...@iglou.com> wrote:
: > "No center to flee from", is that a technical term?

: If you like ... just a literal translation!

Translation from what, Newton's Latin?

: >What can you have against the word "centrifugal"?

: Centrifugal effects don't exist from where I'm looking!

Centrifugal "effects" certainly do exist,
while "centrifugal acceleration" certainly does not.

: > I think I see the problem, in the case of binary


: > stars, you assume a centripetal force, and call it gravity.

: Not quite ... I see gravity and this provides the centripetal force.

I think I understand gravity quite well,
and I assure you that nothing is accelerating toward
the center, and no force is acting on freefalling
objects (Yes, I know, you are unable to consider
anything but Newtonian gravitation).

: > I think Einstein would have disagreed.

: Yes I understand that.

Then why not accomodate something other
than Newtonian gravitation, just acknowledging
the fact does not mean you have to do the math.

: > Orbiting objects are in inertial motion, that is


: > what non-acceleration means, and the General Theory of
: > Relativity is based on that.

: From where I'm looking orbital objects are continuously being accelerated.

Take a look at video from Mir or the Shuttle,


it is pretty obvious the occupants do not feel any
acceleration, and I don't care to hear about gravity
pulling and centrifugal force being equal, I don't
like physics with the same terms on both sides of the
formulas.

: > You keep talking about "centripetal acceleration"


: > in the case of the string and stone.
: >If an automobile goes around a curve, is it "centripetal", or simply
: > "lateral"?

: The friction between the road and the tyre provides a centripetal force
: which causes the car to accelerate, describing a circular path OR continue
: in a striaght line into that tree if the frictional force is removed ...
: say ... by a patch of water!

If the turning angle varies, you then have to have
a moving center point just to be able to retain the concept
of center. This is really stretching to preserve the
"centripetal" aspect as opposed to merely a lateral
acceleration.

: Both centripetal and centrifugal have been defined in a non-rottaing

: frame and must be redifined or preferably not used elsewhere.

Are you an English major teaching physics? :-)
I really think you have to add a qualifier to both
words, and "centrifugal effect" is acceptable as an
"inertial effect", and so is "centripetal force" just
as acceptable to explain what is working against
inertia.
But I don't think it reflects the truth to
use "acceleration" with either word.

Ken Fischer

mahipa...@orbital.fsd.com

unread,
May 20, 1997, 3:00:00 AM5/20/97
to

In article <shepard-1605...@macrls.tcg.anl.gov>,

she...@tcg.anl.gov (Ron Shepard) wrote:
>
> In article <8637311...@dejanews.com>, mahipa...@orbital.fsd.com wrote:
>
> [...]
> >I'm sorry if I seem like "I must sit you down and set you straight", but
> >i am just being polite by letting you know you are in error. What you've
> >said, now twice, is as wrong as it was the first time.
> >
> >The *rotating* reference frame gives rise to the centrifugal force. A
> >frame can be rectangular and still be rotating. A frame can be polar and
> >not be rotating. [...]
>
> This is true, but what rotating frame has been defined that leads to
> centrifugal force?

I think I am begining to both understand and share your confusion. Sorry
if I am inadvertently adding to it. But I keep in mind that we're both
learning from this to-and-fro exchange.

I guess we have two distinct types of experimental scenarios to consider.
One is the twirling your kid by the arms and the second is a Shuttle in
orbit (no strings attached basically).

The question: Where fort art thou "centrifugal force" in each experiment?

[trim Lagrangian derivative wrt r approach discussion]

> >By your argument, all centrifugal forces would simply vanish if we all
> >opted to just use cartesian coordinates.
>
> In a way, yes. It depends on how the derivatives of the potential
> function are defined. It is these derivatives that define the forces,
> right? It also depends on moving that piece of kinetic energy over into
> the potential energy when you take the derivatives.

No, that simply can't be. Forces, centrifugal or otherwise, can't vanish
simply because we analysts prefer to work in cartesian coordinates over
polar ones. That's like believing poverty or sunshine vanishes if we
simply discuss the subject in French as opposed to our lovely English.
[...]

>
> I've heard another story about Dirac notation. Brassieres were first

[...]


> newfangled undergarmets that were just beginning to be called "bras".

You know Ron, both you and I are in the wrong business. We should be
designers for ladies undergarments. Also, I'll bet you anything Dirac's
favorite bra size was a "D == |>" Cup. Dirac, the sexual pervert! I think
my respect and admiration for him just shot up a few notches.

[trim]


> >Basically, the same centrifugal force which exists in the polar reference
> >frame also exists (i.e., can be represented) equivalently in the
> >rectangular frame. This force also exists, however fictional, in an
> >infinity of other equiExistent but not used coordinate bases! The
> >"fictional" is indeed the tricky part.
>
> But if you differentiate the potential w.r.t. the |e_x> and |e_y>
> components, there is indeed no "centrifugal force" that shows up. It is
> when you represent it in terms of r and theta, and add in the conserved
> L^2 contribution from the kinetic energy into the potential, and then
> differentiate w.r.t. "r" that the "centrifugal force" looks like a force.
>
> But the vector transformations are just a tangent (an unintentional pun
> :-) to the original question about the nature of "centrifugal force".
> Does it arise from a rotating coordinate system, or from the choice of a
> polar coordinate system? I've pointed out that the usual textbook
> derivation uses the latter approach. There are of course equivalent
> representations of this force in other coordinate systems, including
> rotating ones, but a rotating coordinate system is not necessary to
> understand, or to derive in a simple way, the definition of this fictional
> "centrifugal force".

I've read your last sentence many many times. Still, I'm confused.
Because, in my opinion, a "rotating coordinate system" can be anything
from rectangular to polar. Would you agree that a "centrifugal force"
arises in cases of particles experiencing rotating==circular motion?
Motion like the two specific examples we have at the top of this post.
Would it be fair to discuss those case examples juxtaposed next to one
another? If not, I'm open to suggestions.

Btw, you and I have to accept that there are many textbook approaches.
Despite the preferred individual approaches of the many authors, the
Physics ought remain invariant under authorial transformations. Call it
the Authorial Transformation Principle. There is a relativistic version
of the ATI but ... ;-)

me...@cars3.uchicago.edu

unread,
May 20, 1997, 3:00:00 AM5/20/97
to

In article <8641360...@dejanews.com>, mahipa...@orbital.fsd.com writes:
>In article <shepard-1605...@macrls.tcg.anl.gov>,

>
>You know Ron, both you and I are in the wrong business. We should be
>designers for ladies undergarments. Also, I'll bet you anything Dirac's
>favorite bra size was a "D == |>" Cup.

Correction, "D = <|" cup. The one above is ket:-)

>Dirac, the sexual pervert! I think my respect and admiration for him
>just shot up a few notches.

I'll second that.

Kris Jirapinyo

unread,
May 20, 1997, 3:00:00 AM5/20/97
to

Hello,
This is out of the subject but I think there is NO SUCH THING AS A
CENTRIFUGAL FORCE! Don't you agree? The only people who calls such a
thing must have a misconception. The only force acting on an object
traveling in uniform circular path is the CentriPETAL force (In space)
which is directed towards the center of the circular path. Therefore,
the acceleration also is directed towards the center of the path. The
velocity, however, is tangent to the circular path. So, when somebody
feels like there is a centrifugal force acting on them when they go ride
something that has a circular path in an amusement path, the centrifugal
force does not really act on them. They want to go straight forward out
of the circular path because of inertia and velocity, but they can't
because there is something that is changing their direction all the
time.

Maybe I am wrong. What do you think?

KJ the physics whiz (not really.)

Mike Norris

unread,
May 21, 1997, 3:00:00 AM5/21/97
to

In article <EAEw6...@iglou.com>, Ken Fischer

<URL:mailto:kefi...@iglou.com> wrote:
>
> Mike Norris (mi...@mikenorr.demon.co.uk) wrote:
> : Ken Fischer (kefi...@iglou.com> wrote:
> : > "No center to flee from", is that a technical term?
>
> : If you like ... just a literal translation!
>
> Translation from what, Newton's Latin?

Yes from Latin, but Latin is Latin ... what's it got to do with Newton?

> : >What can you have against the word "centrifugal"?
>
> : Centrifugal effects don't exist from where I'm looking!
>
> Centrifugal "effects" certainly do exist,
> while "centrifugal acceleration" certainly does not.

No they don't ... you can't have one without the other!

> : > I think I see the problem, in the case of binary
> : > stars, you assume a centripetal force, and call it gravity.
>
> : Not quite ... I see gravity and this provides the centripetal force.
>
> I think I understand gravity quite well,
> and I assure you that nothing is accelerating toward
> the center, and no force is acting on freefalling
> objects (Yes, I know, you are unable to consider
> anything but Newtonian gravitation).

This is just nonsense ...
are you saying that a freefalling object is NOT accelerating ...
... it is so easily measured 9.81m/s2 at the earth's surface!

By the way I'm NOT a fan of Newton!
In spite of the outstanding success of Newtonian Mechanics this century.
I see his ideas and the way he propogated them, crushing the ideas and
theories of others, as holding back the boundaries of Physics even today!

> : > I think Einstein would have disagreed.
>
> : Yes I understand that.
>
> Then why not accomodate something other
> than Newtonian gravitation, just acknowledging
> the fact does not mean you have to do the math.

Yes I do accept other models but they have no bearing on this issue.

> : > Orbiting objects are in inertial motion, that is
> : > what non-acceleration means, and the General Theory of
> : > Relativity is based on that.
>
> : From where I'm looking orbital objects are continuously being accelerated.
>
> Take a look at video from Mir or the Shuttle,
> it is pretty obvious the occupants do not feel any
> acceleration, and I don't care to hear about gravity
> pulling and centrifugal force being equal, I don't
> like physics with the same terms on both sides of the
> formulas.

Why should they when all of their local system is doing the same thing accelerating!
You keep using the term centirfugal to try to explain itself!


> : > You keep talking about "centripetal acceleration"
> : > in the case of the string and stone.
> : >If an automobile goes around a curve, is it "centripetal", or simply
> : > "lateral"?
>
> : The friction between the road and the tyre provides a centripetal force
> : which causes the car to accelerate, describing a circular path OR continue
> : in a striaght line into that tree if the frictional force is removed ...
> : say ... by a patch of water!
>
> If the turning angle varies, you then have to have
> a moving center point just to be able to retain the concept
> of center. This is really stretching to preserve the
> "centripetal" aspect as opposed to merely a lateral
> acceleration.

NOt so ... the radius of the curve gets greater ... eventually infinite
as the friction force decreases to zero.

> : Both centripetal and centrifugal have been defined in a non-rottaing
> : frame and must be redifined or preferably not used elsewhere.
>
> Are you an English major teaching physics? :-)

Absolutely NOT ... just a Physicist who enjoys the use of language
as a tool for promulgation of the subject!

> I really think you have to add a qualifier to both
> words, and "centrifugal effect" is acceptable as an
> "inertial effect", and so is "centripetal force" just
> as acceptable to explain what is working against
> inertia.
> But I don't think it reflects the truth to
> use "acceleration" with either word.

What is the truth?

Essentially there isn't any ... what you observe in from different frames will
be different. Even the so called Laws of Physics vary from frame to frame.

There are only models.

I am simply saying that the use of the term centrifugal is inappropriate in any model!

Mike Norris

unread,
May 21, 1997, 3:00:00 AM5/21/97
to

In article <EAJpM...@midway.uchicago.edu>,
<URL:mailto:me...@cars3.uchicago.edu> wrote:
>
> In article <ant21192...@mikenorr.demon.co.uk>, Mike Norris <mi...@mikenorr.demon.c
> o.uk> writes:
> >In article <EAEHv...@midway.uchicago.edu>,
> ><URL:mailto:me...@cars3.uchicago.edu> wrote:
> >>
> >> In article <ant18193...@mikenorr.demon.co.uk>, Mike Norris <mi...@mikenorr.demo
> n.c

> >> o.uk> writes:
> >> >
> >> >Both centripetal and centrifugal have been defined in a non-rottaing
> >> >frame and must be redifined or preferably not used elsewhere.
> >>
> >> This is wrong.
> >
> >No it isn't. These terms were in use long before anyone even thought
> >of transposing between frames or even had an inkling that there were
> >other places to view from!
>
> These terms have a clear and well defined meaning within mechanics.
> If you intend to talk about physics, use the terms the way they are
> defined in physics. If you intend to talk about the history of
> language, this is not the appropriate newsgroup.

The last sentence is not an appropriate comment for this news group!

> >Why not simply redefine Newtons 3rd Law in the rotating frame so that things
> >work without the need to invent ficticious forces?
>
> Because it is way more convenient to use the same laws everywhere.

That is a good reason but not necessarily all pervading!

Anyway ... how the dickens do you manage to reply so quickly ... and
to so many threads ............... genuine interest ...... not facetious.

me...@cars3.uchicago.edu

unread,
May 21, 1997, 3:00:00 AM5/21/97
to

In article <ant21222...@mikenorr.demon.co.uk>, Mike Norris <mi...@mikenorr.demon.co.uk> writes:
>In article <EAJpM...@midway.uchicago.edu>,
><URL:mailto:me...@cars3.uchicago.edu> wrote:
>>
>> In article <ant21192...@mikenorr.demon.co.uk>, Mike Norris <mi...@mikenorr.demon.c
>>
>> These terms have a clear and well defined meaning within mechanics.
>> If you intend to talk about physics, use the terms the way they are
>> defined in physics. If you intend to talk about the history of
>> language, this is not the appropriate newsgroup.
>
>The last sentence is not an appropriate comment for this news group!

We'll have to agree to disagree on this one.


>
>> >Why not simply redefine Newtons 3rd Law in the rotating frame so that things
>> >work without the need to invent ficticious forces?
>>
>> Because it is way more convenient to use the same laws everywhere.
>
>That is a good reason but not necessarily all pervading!

Of course not. But in the absence of a divine authority which can
provide us with all pervading reasons, we do what's convenient until
it ceases to be convenient.


>
>Anyway ... how the dickens do you manage to reply so quickly ... and
>to so many threads ............... genuine interest ...... not facetious.

I'm next to the computer most of the day, so I check what's new on the
newsgroup anytime I get tired with whatever it is I'm supposed to be
doing (which is quite often). Hey, it is a good excuse to
procrastinate.

me...@cars3.uchicago.edu

unread,
May 21, 1997, 3:00:00 AM5/21/97
to

In article <33835D...@geocities.com>, Kris Jirapinyo <piano...@geocities.com> writes:

>Mike Norris wrote:
>>
>>
>> Why not simply redefine Newtons 3rd Law in the rotating frame so that things
>> work without the need to invent ficticious forces?
>>
>> mi...@mikenorr.demon.co.uk
>> The English Lake District.
>
>I agree with mike here. Whoever thinks that centrifugal and centripetal
>forces are interchangeable or that centrifugal force exists is deeply
>MISTAKEN!!!!!!! There is no such thing and you can't argue with it!
>Maybe somebody had a bad teacher! It is true that they mistakenly use
>it in fiction, but in real world, everybody knows that there is no such
>thing!!!!!!
>
Yeah, you're right. Somebody here really had a bad teacher. Either
this or somebody is trolling.

Mike Norris

unread,
May 21, 1997, 3:00:00 AM5/21/97
to

In article <EAEHv...@midway.uchicago.edu>,
<URL:mailto:me...@cars3.uchicago.edu> wrote:
>
> In article <ant18193...@mikenorr.demon.co.uk>, Mike Norris <mi...@mikenorr.demon.c

> o.uk> writes:
> >
> >Both centripetal and centrifugal have been defined in a non-rottaing
> >frame and must be redifined or preferably not used elsewhere.
>
> This is wrong.

No it isn't. These terms were in use long before anyone even thought of transposing
between frames or even had an inkling that there were other places to view from!

Why not simply redefine Newtons 3rd Law in the rotating frame so that things

me...@cars3.uchicago.edu

unread,
May 21, 1997, 3:00:00 AM5/21/97
to

In article <ant21192...@mikenorr.demon.co.uk>, Mike Norris <mi...@mikenorr.demon.co.uk> writes:
>In article <EAEHv...@midway.uchicago.edu>,
><URL:mailto:me...@cars3.uchicago.edu> wrote:
>>
>> In article <ant18193...@mikenorr.demon.co.uk>, Mike Norris <mi...@mikenorr.demon.c
>> o.uk> writes:
>> >
>> >Both centripetal and centrifugal have been defined in a non-rottaing
>> >frame and must be redifined or preferably not used elsewhere.
>>
>> This is wrong.
>
>No it isn't. These terms were in use long before anyone even thought
>of transposing between frames or even had an inkling that there were
>other places to view from!

These terms have a clear and well defined meaning within mechanics.

If you intend to talk about physics, use the terms the way they are
defined in physics. If you intend to talk about the history of
language, this is not the appropriate newsgroup.
>

>Why not simply redefine Newtons 3rd Law in the rotating frame so that things
>work without the need to invent ficticious forces?

Because it is way more convenient to use the same laws everywhere.

Mati Meron | "When you argue with a fool,

Ron Shepard

unread,
May 21, 1997, 3:00:00 AM5/21/97
to

In article <8641360...@dejanews.com>, mahipa...@orbital.fsd.com wrote:

[...]


>> >By your argument, all centrifugal forces would simply vanish if we all
>> >opted to just use cartesian coordinates.
>>
>> In a way, yes. It depends on how the derivatives of the potential
>> function are defined. It is these derivatives that define the forces,
>> right? It also depends on moving that piece of kinetic energy over into
>> the potential energy when you take the derivatives.
>

>No, that simply can't be. Forces, centrifugal or otherwise, can't vanish
>simply because we analysts prefer to work in cartesian coordinates over
>polar ones. That's like believing poverty or sunshine vanishes if we
>simply discuss the subject in French as opposed to our lovely English.

I think that this is only partly true. There seem to be some forces that
cannot be defined away, some that can, and some that are fictional in
nature that come into being just through some slight of hand in the
defintion. "Centrifugal force" is of the last type. There is a potential
U(r,theta) that defines the "real" potential, and there is that
L^2/(2*mu*r^2) term that gets added to it. In a central field problem
such as gravity between two rigid bodies, U(r,theta) is really only a
function of "r", U(r). The "force" is defined (in this conservative
system) as the derivative of the potential. If you just consider the
original U(r) part, then that is the force between two bodies due to
gravity. But if you include the L^2 part, which is "reasonable" in this
case because L^2 is a conserved quantity, then there is an additional
r-dependent piece that is called "centrifugal force". This turns out to
be mu*r*theta_dot^2, or at least that is one way to write it.

It is perhaps as much of a choice as to whether we discuss this in French
or English, except I must admit I would make very slow progress since I
would have to flip through a French dictionary every other word or so.
:-) Perhaps a better analogy is not whether poverty or sunshine vanish,
but rather whether rhythm and rhyme vanish, or appear, when we change
between French and English.

The choice is also related to what is called the potential energy: is it
just U(r), or is it U(r)+L^2/(2*mu*r^2). That last piece could just as
easily be called "centrifugal kinetic energy" or something, and then there
might be no reason to define the "centrifugal force".

All of these considerations apply to a nonrotating coordinate system.

[...]


>I've read your last sentence many many times. Still, I'm confused.
>Because, in my opinion, a "rotating coordinate system" can be anything
>from rectangular to polar.

I agree, but I have not used either kind of rotating coordinate system, so
my argument is that such a rotating coordinate system is unnecessary to
define, or to understand, "centrifugal force".

>Would you agree that a "centrifugal force"
>arises in cases of particles experiencing rotating==circular motion?
>Motion like the two specific examples we have at the top of this post.
>Would it be fair to discuss those case examples juxtaposed next to one
>another? If not, I'm open to suggestions.

It is not the specific circular motion of the particles that causes
"centrifugal force" to arise, it is the choice of coordinate system, along
with the choice of the definition of the potential energy as discussed
above. It applies not only to circular motion, but also to elliptical
orbits, parabolic orbits, and hyperbolic orbits. I guess it applies also
to the case where U(r)=0, and the trajectory is a straight line, but for
some reason a polar coordinate system was used and the L^2 piece of the
kinetic energy was treated as an effective centrifugal potential.

>Btw, you and I have to accept that there are many textbook approaches.
>Despite the preferred individual approaches of the many authors, the
>Physics ought remain invariant under authorial transformations. Call it
>the Authorial Transformation Principle. There is a relativistic version
>of the ATI but ... ;-)

I agree with this of course. It is not a question of whether "forces" can
be neglected, it is just a matter of whether they are called forces or
not. In other words, it is not only F=ma that is the question, it is also
F=Derivative(U), and whether things that are on the "ma" side of the
equation get shifted over to the Derivative(U) side.

The introduction of rotating coordinate systems is yet another way to try
to simplify some problems. In this case, both "centrifugal force" and
"coriolis force" are introduced so that an equation that looks like F=ma
is satisfied in this coordinate system. Of course, this complicates many
other things, but there is always some kind of tradeoff like this. But a
rotating coordinate system is not _necessary_ for the understanding of
"centrifugal force"; that can be done just fine in an inertial,
nonrotating, coordinate system. Also, there are many things in SR and GR
that depend on rotating and translating coordinate systems, red shifts,
time dilations, length contractions, and so on, but "centrifugal force" is
not one of these things either.

$.02 -Ron Shepard

Mike Norris

unread,
May 21, 1997, 3:00:00 AM5/21/97
to

In article <shepard-2105...@macrls.tcg.anl.gov>, Ron Shepard

<URL:mailto:she...@tcg.anl.gov> wrote:
>
> In article <8641360...@dejanews.com>, mahipa...@orbital.fsd.com wrote:
>
> [...]
> >> >By your argument, all centrifugal forces would simply vanish if we all
> >> >opted to just use cartesian coordinates.

snip

>
> The introduction of rotating coordinate systems is yet another way to try
> to simplify some problems. In this case, both "centrifugal force" and
> "coriolis force" are introduced so that an equation that looks like F=ma
> is satisfied in this coordinate system. Of course, this complicates many
> other things, but there is always some kind of tradeoff like this. But a
> rotating coordinate system is not _necessary_ for the understanding of
> "centrifugal force"; that can be done just fine in an inertial,
> nonrotating, coordinate system. Also, there are many things in SR and GR
> that depend on rotating and translating coordinate systems, red shifts,
> time dilations, length contractions, and so on, but "centrifugal force" is
> not one of these things either.
>
> $.02 -Ron Shepard

This last bit is right.
Newtons Laws of Motion are different in a rotating coordinate system.
It is only necessary to invent centrifugal force to overcome the limitation of not
transforming Newtons Laws to the new frame!

Kris Jirapinyo

unread,
May 21, 1997, 3:00:00 AM5/21/97
to

Mike Norris wrote:
>
> In article <EAEHv...@midway.uchicago.edu>,
> <URL:mailto:me...@cars3.uchicago.edu> wrote:
> >
> > In article <ant18193...@mikenorr.demon.co.uk>, Mike Norris <mi...@mikenorr.demon.c
> > o.uk> writes:
> > >
> > >Both centripetal and centrifugal have been defined in a non-rottaing
> > >frame and must be redifined or preferably not used elsewhere.
> >
> > This is wrong.
>
> No it isn't. These terms were in use long before anyone even thought of transposing
> between frames or even had an inkling that there were other places to view from!
>
> Why not simply redefine Newtons 3rd Law in the rotating frame so that things
> work without the need to invent ficticious forces?
>
> mi...@mikenorr.demon.co.uk
> The English Lake District.

I agree with mike here. Whoever thinks that centrifugal and centripetal


forces are interchangeable or that centrifugal force exists is deeply
MISTAKEN!!!!!!! There is no such thing and you can't argue with it!
Maybe somebody had a bad teacher! It is true that they mistakenly use
it in fiction, but in real world, everybody knows that there is no such
thing!!!!!!

Fc= centripetal force which is directed towards the center
Ac= centripetal acceleration which is also directed towards the center
v= velocity which is tangent to the circular path

Get it right!

With concern,
KJ

Ken Fischer

unread,
May 22, 1997, 3:00:00 AM5/22/97
to

Mike Norris (mi...@mikenorr.demon.co.uk) wrote:

: Ken Fischer kefi...@iglou.com> wrote:
: > Mike Norris (mi...@mikenorr.demon.co.uk) wrote:
: > : Ken Fischer (kefi...@iglou.com> wrote:
: > : > "No center to flee from", is that a technical term?
: >
: > : If you like ... just a literal translation!
: >
: > Translation from what, Newton's Latin?
:
: Yes from Latin, but Latin is Latin ... what's it got to do with Newton?

You are pulling my leg, aren't you? I think
everything I have ever read written by Newton was in Latin,
and had to be translated for me to read it.

: > : >What can you have against the word "centrifugal"?


: >
: > : Centrifugal effects don't exist from where I'm looking!
: >
: > Centrifugal "effects" certainly do exist,
: > while "centrifugal acceleration" certainly does not.

: No they don't ... you can't have one without the other!

I think anybody that needs to know, does know
that centrifugal effects are inertial effects, they are
not "fictitious" in the sense that they don't exist,
inertial forces (resistance to change of motion) do exist.

: > : > I think I see the problem, in the case of binary


: > : > stars, you assume a centripetal force, and call it gravity.
: >
: > : Not quite ... I see gravity and this provides the centripetal force.
: >
: > I think I understand gravity quite well,
: > and I assure you that nothing is accelerating toward
: > the center, and no force is acting on freefalling
: > objects (Yes, I know, you are unable to consider
: > anything but Newtonian gravitation).

: This is just nonsense ...
: are you saying that a freefalling object is NOT accelerating ...

: ... it is so easily measured /s2 at the earth's surface!

Apparently you haven't read the first chapter
of "Gravitation" by Misner, Thorne and Wheeler.

: By the way I'm NOT a fan of Newton!


: In spite of the outstanding success of Newtonian Mechanics this century.
: I see his ideas and the way he propogated them, crushing the ideas and
: theories of others, as holding back the boundaries of Physics even today!

I see nothing wrong with Newtonian mechanics,
it is a subset of whatever the actual underlying physics
might be.

: > : > I think Einstein would have disagreed.


: >
: > : Yes I understand that.
: >
: > Then why not accomodate something other
: > than Newtonian gravitation, just acknowledging
: > the fact does not mean you have to do the math.

: Yes I do accept other models but they have no bearing on this issue.

They do if you insist that an orbiting body
is accelerating, or that a freefalling body is accelerating,
because then gravity is the centripetal force and the only
thing resisting that centripetal force would be inertial
effects, which makes mechanics too simple, but I won't
go anyy farther into that here.

: > : > Orbiting objects are in inertial motion, that is


: > : > what non-acceleration means, and the General Theory of
: > : > Relativity is based on that.
: >
: > : From where I'm looking orbital objects are continuously being
: > : accelerated.
: >
: > Take a look at video from Mir or the Shuttle,
: > it is pretty obvious the occupants do not feel any
: > acceleration, and I don't care to hear about gravity
: > pulling and centrifugal force being equal, I don't
: > like physics with the same terms on both sides of the
: > formulas.

: Why should they when all of their local system is doing the same
: thing accelerating!
: You keep using the term centirfugal to try to explain itself!

No, in the case of gravity, any freefalling or
orbiting object experiences _no_ forces. Any physicist
that can't see that needs to study Einstein.

: > : > You keep talking about "centripetal acceleration"


: > : > in the case of the string and stone.
: > : >If an automobile goes around a curve, is it "centripetal", or simply
: > : > "lateral"?
: >
: > : The friction between the road and the tyre provides a centripetal force
: > : which causes the car to accelerate, describing a circular path OR
: > : continue
: > : in a striaght line into that tree if the frictional force is removed

: > : say ... by a patch of water!


:
: > If the turning angle varies, you then have to have
: > a moving center point just to be able to retain the concept
: > of center. This is really stretching to preserve the
: > "centripetal" aspect as opposed to merely a lateral
: > acceleration.
:
: NOt so ... the radius of the curve gets greater ... eventually infinite
: as the friction force decreases to zero.

You are carrying this too far, not only can the
radius of the curve change in the case of aircraft and
spacecraft, but the direction of change of motion can
change also, so then the "center fleeing" concept gets
lost.
You would have to see the 3 axes rig that
the early astronauts trained in to appreciate the
complexities of free motion and inertial effects.

: > : Both centripetal and centrifugal have been defined in a non-rottaing

: > : frame and must be redifined or preferably not used elsewhere.

: >
: > Are you an English major teaching physics? :-)

: Absolutely NOT ... just a Physicist who enjoys the use of language
: as a tool for promulgation of the subject!

I see nothing gained by trying to remove the
word "centrifugal" from the language, and a lot lost
if the word could be removed.

: > I really think you have to add a qualifier to both


: > words, and "centrifugal effect" is acceptable as an
: > "inertial effect", and so is "centripetal force" just
: > as acceptable to explain what is working against
: > inertia.
: > But I don't think it reflects the truth to
: > use "acceleration" with either word.

: What is the truth?

Simply that centripetal meand "toward the center",
and centrifugal means "away from the center".
So it is acceptable to say "centrifugal effects"
but not "centrifugal acceleration".
It would be more correct to say that what is
called "centrifugal" is really simply an attempt by
moving objects to continue moving in a straight line.
But then "centripetal" is not really correct
to describe something causing those objects to depart
from rectilinear motion.

: Essentially there isn't any ... what you observe in from different frames

: will
: be different. Even the so called Laws of Physics vary from frame to frame.
:
: There are only models.
:
: I am simply saying that the use of the term centrifugal is inappropriate
: in any model!

And here I thought the laws of physics hold the
same anywhere and everywhere.
Some models are better than others.

I give up, but please tell me what to call a centrifuge.

Ken Fischer

Mike Norris

unread,
May 22, 1997, 3:00:00 AM5/22/97
to

> Yeah, you're right. Somebody here really had a bad teacher. Either
> this or somebody is trolling.

Nah ... somebody here has lost their humour!

Mike Norris

unread,
May 22, 1997, 3:00:00 AM5/22/97
to

In article <EAKI4...@iglou.com>, Ken Fischer
<URL:mailto:kefi...@iglou.com> wrote:

> I give up, but please tell me what to call a centrifuge.
>
> Ken Fischer

I've covered that one already ... how about ...

a device that contributes a lack of centripete to suspended particles!!!!!

or as Stanley Unwin would say ....

lackycentipetosupensiparticlesmachine

mahipa...@orbital.fsd.com

unread,
May 23, 1997, 3:00:00 AM5/23/97
to

In article <EAE9F...@midway.uchicago.edu>,
me...@cars3.uchicago.edu wrote:
>
> In article <8639853...@dejanews.com>, mahipa...@orbital.fsd.com writes:
[...]

>
> >I am certain you and I agree here almost exactly. Newton was a very
> >capable physicist for _his_ time. If Newton were alive today, he might
> >not make the grade!
>
> Frankly, I doubt it. Seeing how much Newton managed to do with what
> he had available, I'm reasonably sure that would he live nowadays, his
> contribution would've been enormous. Different, mind you, but
> enormous. Still, arguing over what might've been is pretty pointless.

I doubt it too. I was hoping to personify Newton as "mortal" and not
beyond accessibility. Today wouldn't Newton be lost mastering Doom?!

Where's your romantic side? "Might've beens" are graduate level "What
ifs". One has to be cautious about which ones to consider/pursue.

[trim]

> >It's not "my" statement. It's a reflection of what readers are lead to
> >believe when they read some advanced, and some not so advanced, physics
> >literature. For reference: "Gravity is not a force." I'm sure many have
> >read this before.
>
> Sure. Now. compare the two statements "Mahipal isn't an aardvark" and
> "there are no aardvarks in the world". Do you see a slight difference
> between them?

What I wouldn't give to nonexist. Err. Scratch that. Nevermind. :-)
I like my existence, whatever its actual |state> and or ||worth||.

To your point, I do think I can recognize weak syllogisms. I hope.

> So, one should be a bit careful before jumping to broad sweeping
> generalizations based on limited statements. Gravity is not the only
> thing around, you know. Beyond this, one should be careful
> interpreting statements like "X is (or isn't) a force". Their meaning
> depends on how broad or narrow our definition of a force is.
> Interestingly enough this ties back into the issue of centrifugal
> forces which started this debate. As i wrote elsewhere, we
> distinguish between "real physical forces" which can be ultimately
> traced to some particle-particle interaction and "inertial forces"
> which result from the selection of a specific reference frame to
> describe the dynamics and can be fully eliminated by choosing another
> reference frame. Mind you, those "inertial forces" still have very
> real effects relative to frames where they're not eliminated. So,
> anyway, when we say "gravity is not a force" what we mean is "gravity
> belongs in the inertial forces category, not in the real physical
> forces one".

Now that's a semantic quibble! Thanks for the clarification. But I'm
still confused between what my inconsistent textbooks state --- and with
all good cheer and well placed respect --- what some guy on Usenet just
said. ;-)

Btw, I think there's a Newman metric which already accounts for the
charges of particles in the GR formulation. It basically adds in an
additional term for the Coulomb Potential.

me...@cars3.uchicago.edu

unread,
May 23, 1997, 3:00:00 AM5/23/97
to

In article <8644078...@dejanews.com>, mahipa...@orbital.fsd.com writes:
>In article <EAE9F...@midway.uchicago.edu>,
>> Frankly, I doubt it. Seeing how much Newton managed to do with what
>> he had available, I'm reasonably sure that would he live nowadays, his
>> contribution would've been enormous. Different, mind you, but
>> enormous. Still, arguing over what might've been is pretty pointless.
>
>I doubt it too. I was hoping to personify Newton as "mortal" and not
>beyond accessibility. Today wouldn't Newton be lost mastering Doom?!
>
Alas, this is not unthinkable:-)

... snip ...

>>As i wrote elsewhere, we
>> distinguish between "real physical forces" which can be ultimately
>> traced to some particle-particle interaction and "inertial forces"
>> which result from the selection of a specific reference frame to
>> describe the dynamics and can be fully eliminated by choosing another
>> reference frame. Mind you, those "inertial forces" still have very
>> real effects relative to frames where they're not eliminated. So,
>> anyway, when we say "gravity is not a force" what we mean is "gravity
>> belongs in the inertial forces category, not in the real physical
>> forces one".
>

>Now that's a semantic quibble! Thanks for the clarification. But I'm
>still confused between what my inconsistent textbooks state --- and with
>all good cheer and well placed respect --- what some guy on Usenet just
>said. ;-)
>

Being confused is fine. It is when you're absolutely certain, that
you're heading for trouble.

>Btw, I think there's a Newman metric which already accounts for the
>charges of particles in the GR formulation. It basically adds in an
>additional term for the Coulomb Potential.

Since you still won't have two particles, one positive the other
negative, following the same trajectory, this is pretty much useless.

Oeandrr

unread,
May 24, 1997, 3:00:00 AM5/24/97
to

We always refer to it as centrifugal effect, not force.

Jim Carr

unread,
May 27, 1997, 3:00:00 AM5/27/97
to

Kris Jirapinyo <piano...@geocities.com> writes:
>
>I agree with mike here. Whoever thinks that centrifugal and centripetal
>forces are interchangeable or that centrifugal force exists is deeply
>MISTAKEN!!!!!!!

Two different things here. They are not interchangable. That way
lies the madness of the mule that cannot put its cart because of
equal and opposite forces. You must keep track of what force
acts on what body. The following, from another article, is quite
correct:

} The only force acting on an object
}traveling in uniform circular path is the CentriPETAL force (In space)

}which is directed towards the center of the circular path.

Now ask yourself what body is applying that force on the object.
Newton's third law says that when the body applies this force, an
equal and opposite force is applied to it. What direction is that
force? It is center-fleeing, centrifugal. That body does not
accelerate because of other forces on it, of course, so there is
no centrifugal acceleration due to that force.

Thus your other earlier remark

}the centrifugal force does not really act on them.

is on the money. There is a centrifugal force, but it does not
act on the object experiencing the centripetal acceleration. This
was stated correctly in the dictionary definition that started
this thread.

>It is true that they mistakenly use
>it in fiction, but in real world, everybody knows that there is no such
>thing!!!!!!

The real world we live in is not an inertial frame. Working with
axes fixed in a laboratory, there are "fictitious" non-inertial
forces as a result. Our weather is due to one of these, and part
of the latitudinal variation in g is due to another.

Try explaining the precession of a Foucalt pendulum at mid-latitudes
and you will discover that non-inertial forces are actually the
easiest way to solve it. They are not evil things to be avoided
at all costs.

--
James A. Carr <j...@scri.fsu.edu> | Commercial e-mail is _NOT_
http://www.scri.fsu.edu/~jac/ | desired to this or any address
Supercomputer Computations Res. Inst. | that resolves to my account
Florida State, Tallahassee FL 32306 | for any reason at any time.

0 new messages